Surface Chemistry and POC PDF

Download as pdf or txt
Download as pdf or txt
You are on page 1of 57

SURFACE CHEMISTRY

Adsorption
The surface of the solid has a tendency to attract the molecules of a gas or a liquid when they
come in its contact. Since energy is released on account of attraction, the solid acquires a more
stable state. These molecules attracted by the finely divided solid, remain on the surface only and
do not penetrate into the bulk of the solid. Thus, there is a high concentration of these species of
gas or liquid on the surface and they are said to be adsorbed on the surface. Hence, adsorption
may be defined as:

The phenomenon of higher concentration of the molecular species


(gases or liquids) on the surface of the solid than in the bulk.

Adsorbent and adsorbate: The solid substance on the surface of which adsorption occurs
is known as adsorbent while the molecular species that get adsorbed on the solid surface due to
intermolecular attractions are called adsorbate.For example, activated charcoal which is finely
divided has a tendency to adsorb a number of gases like ammonia, phosgene etc. Here charcoal
acts as an adsorbent while gas molecules act as adsorbate. In a similar manner, animal charcoal is
used for decolourising a number of organic substances in the form of their solution. The
discharge of the colour is due to fact that the coloured component (generally an organic dye) gets
adsorbed on the surface of the adsorbent i.e. animal charcoal.

Explanation of Adsorption: As discussed above, the adsorption is a surface phenomenon.


The atoms in a solid are held together by certain binding forces which are main van der Waal’s
forces or may be even chemical forces. The unutilized or unbalanced forces present on the
surface (indicated by dotted lines) make it quite active. Moreover, when a solid substance shown
in figure 1 (a) is broken into two or more pieces, new surfaces get formed shown in figure 1 (b)
i.e., the number of the unbalanced forces becomes more. In other words, the number of active
centres or free valencies becomes more by taking the adsorbent in the finely powdered form.

X X X X
+
X X X X

(a) (b)
Figure 1: Unbalanced forces on the finely divided solid surface

Due to the unbalanced forces, the atoms or molecules on the surface have a tendency to attract
and retain the molecules of the gases or liquids when they come in contact with the surface.
results into higher concentration of any particular species at the surface than in the bulk and this
leads to adsorption. The absorbent which are commonly used are activated charcoal, animal
charcoal, silica gel and a number of finely divided metals such as Cu, Ni, Pd, etc. Activated
charcoal is a powerful adsorbent since it is a microscopic crystalline variety of graphite having a
very large surface area.

Desorption:Adsorption is of reversible nature i.e., the species which get adsorbed on the
surface of the adsorbent have a tendency to come out of the surface. The reverse phenomenon is
called desorption. Though the desorption takes place of its own, it can be accelerated by
increasing the temperature. The energy absorbed weakens the forces binding the molecules of the
adsorbent and the adsorbate. As a result, the latter gets released from the surface. The desorption
is very common in gases which are adsorbed on the surface of finely divided metals as a result of
weak van der Waal’s forces of attraction. They can be easily overcome and lead to desorption.

Absorption
Absorption is another important phenomenon which is noticed in our daily life. When the
molecules of a substance are uniformly distributed throughout the body of other substance, it is
known as absorption. For example, when ammonia gas is passed through water, the molecules of
ammonia get uniformly distributed in water and we say that ammonia has been absorbed by
water.

Difference between Adsorption and Absorption

On the basis of the above discussion, it appears that adsorption is quite different from absorption.
Where adsorption is linked only with the surface of the substance which takes up the particles of
the other substance, absorption occurs in the bulk of the substance which absorbs. In order to
make a proper distinction between the two, let us place a silica gel and anhydrous calcium
chloride on contact with water vapours taken in two separate containers. When silica gel is
placed in an environment of water vapours, it simply adsorbs these water vapours. As shown in
figure 2(a), the water vapopurs are present in a high concentration at the surface of silica gel
whereas the interior of the gel has no water vapours. On the other hand, when anhydrous calcium

Water vapours

Anhydrous CaCl2

Silica Gel

Figure 2: (a) Higher concentration of water Figure 2: (b) Uniform distribution of water
vapours at the surface of silica gel vapours throughout the body of calcium
(Phenomenon of a adsorption) chloride (Phenomenon of absorption)

chloride is placed in water vapours, it absorbs these. The water vapours uniformly get distributed
throughout the body of anhydrous calcium chloride as shown in figure 2(b). Thus, silica gel
absorbs water vapours while anhydrous calcium chloride absorbs moisture. It is quite obvious
that the concentration of the water vapours is mainly on the surface of the gel while these
vapours have penetrated into the bulk of the anhydrous calcium chloride. Now, as the adsorption
is linked with the surface only, the extent of adsorption depends upon the availability of space on
the surface. Therefore, it is expected to be quite fast to start with while its rate must decrease with
the passage of time. On the other hand, the absorption must proceed at a uniform speed since it is
no way connected with the surface of the substance which absorbs. The main points of distinction
between the two phenomenon are listed in a tabular form.
Table 1: Distinction between absorption and adsorption
Absorption Adsorption
1. It is phenomenon as a result of which It is the phenomenon as a result of which the
the species of one substance get species of one substance get concentrated
distributed uniformly throughout the mainly at the surface of the other substance
body of the other substance. which adsorbs these.
2. The concentration of the same The concentration on the surface of the
throughout the material. adsorbent is different from that in the bulk.
3. It is a bulk phenomenon since it It is only a surface phenomenon since it occurs
occurs throughout the bulk of the only at the surface.
material.
4. Absorption proceeds at uniform rate. Adsorption is rapid in the beginning and its
rate slowly decreases with the non-availability
of the surface.

Sorption:Some cases are known in which both adsorption and absorption occur together. This
means that a particular substance gets distributed throughout the bulk of the other substance but
at the same time, its concentration on the surface also increases and is more than in the bulk.
However, it is very difficult to make a clear distinction between the two. This is known as
sorption. Certain dyes get adsorbed as well as absorbed on the fibres resulting in sorption.

Thermodynamical Feasibility of Adsorption: We have studied that the phenomenon


adsorption arises because of attraction between the particles or molecules of adsorbent and
adsorbate. Therefore, it is exothermic in nature and is accompanied by release in energy known
as enthalpy of adsorption (heat of adsorption). It may be defined as:

the change in enthalpy taking place for the adsorption of one mole of the
adsorbate on the surface of the adsorbent.

The adsorption is also likely to influence the randomness, (T∆S) factor. The randomness of the
adsorbed species on the surface of the adsorbent is likely to decrease i.e., T∆S is negative. Let us
study the feasibility of the phenomenon in the light of Gibb’s Helmholtz equation. Since ∆H
favours while T∆S opposes the phenomenon of adsorption, the magnitude of the former must be
more. This is feasible only at low temperature and this is what we actually notice. We shall
discuss the effect of temperature on the rate of adsorption at a later stage.
∆G = ∆H − T∆S
( − ve) ( − ve)

Physical and Chemical Adsorption


Adsorption as discussed arises because of the attraction between the particles of the adsorbate
and adsorbent. Depending upon the nature of the forces, it has been classified into physical and
chemical adsorption.

Physical adsorption:When the particles of the adsorbate are held to the surface of the
adsorbent by physical forces such as van der Waal’s forces, it is called physical adsorption. This
is called physisorption or even van der Waal’s adsorption. Since the attractive forces are weak,
they can be easily overcome either by increasing the temperature or decreasing the pressure.
Therefore, the physisorption can be easily reversed or decreased by either raising temperature or
by reducing the pressure of the gas. The enthalpy of adsorption in case of physical adsorption is
quite low. It is varies between 20 to 40 kJ mol–1.

Chemical adsorption:When the molecules of the adsorbate and adsorbent are held together
by stronger forces similar to the chemical bonds, the adsorption is known as chemical adsorption
or Chemisorption. This leads to formation of some sort of compound on the surface of the
adsorbent. These are known as surface compounds. This is further confirmed by the fact that the
when oxygen is adsorbed on the surface of metal like tungsten or activated charcoal, the
molecules which are released from the surface are not of oxygen but those of tungsten oxide in
case of metal and of carbon monoxide or carbon dioxide in case of charcoal. Moreover, the
enthalpy of adsorption unlike physical adsorption is also quite high (nearly 400 kJ mol–1) which
is almost of the same magnitude that we come across in chemical reactions. However, it is mono
layer in nature which means that only the atoms of the molecules present on the surface of the
adsorbent are involved in the bond formation. Physical adsorption, on the other hand is
multilayer in nature because more than one layer of the adsorbent may be involved in van der
Waal’s forces of attraction. Another major difference between the two types of adsorption is that
whereas the physical adsorption can be easily reversed (i.e. desorption is very easy), it is not so in
case of chemical adsorption since bonds are to break and certain energy is needed for the same.

The important points of distinction between the two types of adsorption are given:

Table 2: Difference between Physisorption and Chemisorption


Property Physisorption Chemisorption
1. Nature of forces Weak van der Waal’s forces Strong chemical forces present
present.
2. Heat of adsorption Low; of the order of 20-40kJ/ High; of the order of 400kJ/mol
mol.
3. Temperature Usually occurs at low Occurs at high temperature
temperature
4. Reversibility Reversible Irreversible
5. Specificity It is not specific in nature, i.e., It is highly specific in nature and
all gases are adsorbed on the occurs only when there is bond
surface of a solid to the same formation between adsorbent and
extent. adsorbate molecules.
6. Nature of molecular Forms multi-molecular layers Forms mono-molecular layer
layers
7. Extent of adsorption The extent of adsorption is There is no correlation between
directly related with the ease of extent of adsorption and the ease
liquefaction of the gas. of liquefaction of the gas.
8. Activation energy It does not involve appreciable It generally requires appreciable
activation energy. In most activation energy since a chemical
cases, the activation energy reaction is to take place.
required is almost nil.
9. State of adsorbate Same state as in the bulk. State of adsorbate molecules may
be quite different from that in the
bulk.
Factors Influencing Adsorption of Gases by Solids
Gases are adsorbed on the surface of either finely divided metals such as Ni, Pt, Fe, Pd etc. or
porous solids such charcoal or as activated charcoal, animal charcoal, silica gel, alumina etc. The
extent of adsorption depends upon a number of factors which are briefly discussed.

1. Nature of gas or adsorbate:Since physical adsorption is non specific in nature,


every gas will get adsorbed on the surface of any solid to a lesser or greater extent. The
only criteria for the extent of adsorption is the magnitude of the attractive forces which
exist between the particles of the adsorbent and adsorbate. For example, under given
conditions of temperature and pressure, the easily liquefiable gases like NH3, HCl and
CO2 are adsorbed to greater extent than the gases like H2, O2 and N2 which are liquefied
with difficulty. The former being heteroatomic in nature, have a greater magnitude of the
attractive forces as compared to the latter. In general, the ease of the liquefaction of gases
can be compared in terms of their critical temperatures. The critical temperature of a
gas may be defined as:

the minimum temperature above which a gas cannot be liquefied however large its
pressure may be

Thus greater the value of critical temperature easier will be the ease of liquefaction of the
gas and more will be the extent of its adsorption on a particular solid. The critical
temperatures of different gases along the extent of their adsorption in cc in one gram of
the activate charcoal (adsorbent) are listed.

Gas H2 N2 CO CH4 CO2 HCl NH3 SO2


Critical temperature (K) 33 126 134 190 304 324 406 430
Amount adsorbed (in cc) 4.7 8.0 9.3 16.2 48 72 18 380
 Extent of adsorption increases →
Note: This does not apply to chemisorption since adsorption can occur only when specific bonds are
formed between the molecules of adsorbate and adsorbent. Thus, chemisorption is of selective
nature.

2. Nature of adsorbent:The nature of adsorbent which is normally a solid is linked


with its surface area. Animal charcoal, silica gel and alumina are regarded as good
adsorbents because of their porous nature and large surface area. Metals acting as
adsorbent are also taken in finely divided form so that large surface area may be available
for adsorption of the gases.

3. Activation of the adsorbent:Activation of adsorbent implies increasing its


adsorbing power. This is done by increasing the surface area and also the number of pores
or vacant sites (also called activate centres) per unit area. This can be carried in a number
of ways.
i) Activation of the adsorbent is carried out by mechanical rubbing or by subjecting it to
some chemical reaction.

ii) Some adsorbents are activated by strong heating in contact with superheated steam.
For example, charcoal which is commonly employed in adsorption of gases, is
activated by subjecting it to the action of superheated steam. This will remove the
molecules of gases like CH4, C2H6 (hydrocarbons) that are occupying these pores.

iii) To increase the adsorbing power, the adsorbent is sub-divided into small fine pieces.
By doing so, the surface area increases and so is the adsorbing power.

4. Effect of Pressure:In order to study the effect of pressure on the adsorption of a gas
on the surface of a solid, we must bear in mind that adsorption is a reversible process i.e.,
desorption is also taking place simultaneously. In case, the adsorbate and adsorbent are
enclosed in a closed container, a stage is reached when the amount of gas adsorbed
becomes equal to the amount of gas desorbed. This stage is reached when the amount of
gas adsorbed becomes equal to the amount of gas desorbed. This stage is called
equilibrium state i.e. the surface of the adsorbent gets saturated with the adsorbed gas and
no more adsorption occurs. The extent of the adsorption is generally expressed as x/m
where m is the mass of the adsorbent and x is the mass of the adsorbate when equilibrium
has been attained. If we plot a graph between x/m and the pressure p of the gas at a
constant temperature, the curve is called adsorption isotherm.

A simple type of adsorption isotherm (i.e. the relation


between the pressure of the gas (adsorbate) and the x/m
amount (x/m) of it adsorbed on the surface of
adsorbent is shown in figure 3.

At the equilibrium pressure, ps, x/m reaches its


maximum value i.e., no more adsorption takes place
even if the pressure is further increased. This rate is O P s P
also called saturation state in adsorption and the Figure 3: Adsorption isotherm

corresponding pressure (ps) is regarded as saturation


pressure. Such isotherms are considered when the adsorbate simply forms a unimolecular
layer on the surface of adsorbent.

On the basis of experimental studies, Freundlich gave a relation between the amount of
the gas adsorbed (x) per unit mass of the adsorbment (m) and the pressure (p). This is
known as Freundlich’sadsorption isotherm because the graph has been plotted at
constant temperature.
i) Form the adsorption isotherm (Figure 3), it is clear that at low pressure, the graph is
almost straight. This means that x/m is directly proportional to the pressure. This may
be expressed as
x x
∝ p1 or = K ⋅ p1
m m
Where K is constant.
ii) At high pressure:The value of x/m becomes almost constant, i.e., it does not
change with the change in pressure. The relation may be expressed as:
x x
=constant or ∝ p0 (Q p 0 = 1)
m m
x
or = K p0
m
iii) At intermediate range of pressure, x/m, will depend upon the power of
pressure which lies between 0 to 1 i.e., fraction power of pressure. This may be
expressed as

x x
∝ p1/ n or = K ⋅ p1/ n
m m

Where n is a constant (whole number) which /n


e =1
depends upon the nature of adsorbate and adsorbent. Slo

log x/m
The constants K and n can be determined as follows:
Intercept = log K

Taking logarithsm on both sides of equation, we get O log p


Figure 4: Liner graph between log x/m and log p.

x 1
log = log K + log p
m n

Thus, if we plot a graph between log (x/m) and log p, a straight line will be obtained.
Thus slope of the line (figure 4) is equal to 1/n and the intercept on log (x/m) axis will
correspond to log k.

Langmuir Adsorption Isotherm: This was studied by Langmuir in 1916. The main
features of the adsorption theory are as follows:

i) All adsorption sites on the surface of the adsorbent are equivalent and the ability of a
particle to bind itself to a particular site is quite independent of the presence of other
sites whether occupied or not.

ii) The layer of the adsorbed gas is only one molecule thick i.e., it is unimolecular in
nature. Since this type of adsorption is noticed only in case of chemisorption.
Langmuir adsorption isotherm is applied only to chemisorption.

iii) Adsorption process represents two opposing tendencies i.e., condensation (or
adsorption) of the gas molecules on the surface of the solid and evaporation (or
desorption) of these molecules back to the gaseous state.

iv) The rate of condensation of a gas is proportional to the uncovered surface of the
adsorbent while that of evaporation to its covered surface.

To start with, the entire surface of the solid (adsorbent) is uncovered and the rate of
condensation is very high while that of evaporation is negligible. With the passage of
time, more and more of the surface becomes covered. As a result, the rate of condensation
decreases while that of evaporation increases till the two are equal.

Derivation of Expression for Langmuir Adsorption Isotherm: Let 1 square centimetre


of the surface solid be exposed to a gas maintained at a pressure P. Let at any instant of
the time, the fraction of the surface covered by the adsorbate molecules beθ sq. cm.
Therefore, the uncovered surface of the adsorbent will be (1 – θ) sq. cm.
The rate of condensation ∝ (1 – θ) = k1 (1 – θ)P (1 − θ) sq. cm θ sq. cm

The rate of evaporation ∝θ = k2θ (uncovered) (covered)

At the adsorption equilibrium,


Rate of condensation = Rate of evaporation
k1(1 – θ)p = k2θ or k1p – k1pθ = k2θ
Area = 1 square cm

k1 P
∴ θ(k 2 + k1P) = k1P or θ
(k 2 + k1P)
(k 2 / k 2 P) bP
Dividing by k2; θ = = (here b = k1/k2 = constant)
(1 + k1 / k 2 P) 1 + bP
Now, θ is proportional to x/m i.e. the mass of the gas adsorbed per unit mass of the
adsorbent (solid).
bP k bP aP
∴ x/m∝θ= = 3 =
1 + bP 1 + bP 1 + bP
Here, is a constant and is equal to k3b
aP
Thus, x/m = (Langmuir Adsorption Isotherm)
1 + bp

Validity of the Langmuir Adsorption Isotherm

According to adsorption isotherm.


aP 1 /n
x/m= Sl oe=
θ
a Slope = b/a
1 + bP a
x/m a
or = P
P 1 + bP x/m Intercept = 1/a
Taking reciprocal
P 1 + bP 1 b O P
= = + P Figure 5: Graph between P/ x/m and P in
x/m a a a representing Langmuir Adsorption isotherm

P
Since a and b are constant, a graph between and P must be straight line. The slope
x/m
is equal to b/a and intercept on Y-axis is equal to 1/a.

5. Effect of temperature (Adsorption Isobar):When a gas is not involved in the


formation of any compound with the surface of the adsorbent, it represents chemisorption.
The curves between x/m (amount of the gas adsorbed per unit mass of adsorbent) and the
temperature at constant pressure for the two types of adsorption are as follows:

For physisorption
For physisorption
x/m

x/m

T T
Figure 6: Adsorption isobars for (a) Physical adsorption and (b) Chemisorption
From the curves, it is evident that the extent of adsorption decreases with rise in
temperature in case of physical adsorption or physisorption. This is quite expected also
because adsorption is of exothermic nature. Therefore, the reverse process i.e., desorption
is endothermic in nature. As the temperature is increased, the process desorption i.e.,
reverse process is favoured. In other words, the extent of adsorption of the gas on the
surface of the solid decreases with rise in temperature.

Gas + Solid  
 Gas absorbed; ∆H=-ve

The similar trend is expected in chemisorption also but here the adsorption is initially
shown to increase with the rise in temperature and then follows the same trend. This may
be attributed to the fact that the energy supplied by the initial rise in temperature provides
the molecules of the adsorbate the desired activation energy so that proper bonds could be
formed with the particles of the adsorbent. When all the species of the adsorbate have
been properly oriented, a further rise in temperature will decrease the extent of adsorption
as in case of physical adsorption.

Adsorption from Solution: Apart from adsorption of the gases, solids have also the
capacity to adsorb substances present in solutions. For example, if we place a piece of charcoal in
a litmus solution taken in a test tube and shake, the solution becomes colourless. It is because of
adsorption of the litmus which is in fact a dye, by the charcoal. Similarly, animal charcoal
decolourses impure sugar solution by adsorbing colouring dye.

The actual mechanism of adsorption from the solution is not definite. However, it is believed that
it continues till a unimolecular layer is built up on the surface of the adsorbent. The Freundlich
adsorption isotherm as well as Langmuir adsorption isotherm are applicable to the adsorption
from the solutions in the same way as from the gases. The effect of temperature is also similar.
However, equilibrium concentration (C) is used in place of equilibrium pressure and the
mathematical expressions for the two types of isotherms are:

x
= k ⋅ C1/ n (Freundlich Adsorption Isotherm)
m
x aC
or = (Langmuir Adsorption Isotherm)
m 1 + bC
Completing Adsorption: When more than one type of the adsorbate species are in
contact with a particular adsorbent, there is a competition between their molecules or
particles to be adsorbed on the solid surface. This depends upon their nature. However, the
one which can form stronger bonds is adsorbed more in preference to the other. This is
known as competing adsorption and it also happens in the reverse process also i.e.
desorption. In this case, the species which is weakly adsorbed are dislodged first from the
solid surface. It may be noted that the adsorption chromatography is based upon this
principle of adsorption.

Applications of Adsorption
The phenomenon of adsorption finds extensive applications in industry in laboratory and also in
various chemical processes. A few important applications are listed below:
1. Removal of colouring matter:Many substances such as sugar, juice and vegetable
oils are coloured due to the presence of impurities. They can be decolorized by placing in
contact with adsorbents like activated charcoal or fuller’s earth. Similarly, most of the
organic compounds that are synthesized in the laboratory are coloured. They can also be
decolorised with the help of animal charcoal.

2. In gas masks:Activated charcoal is generally used in gas masks to adsorb poisonous


and toxic gases from air. These masks are used by the miners since there is atmosphere of
CO and other such gases prevailing in the coal mines.

3. In dyeing of cloth:Mordants such as alums are used in dying of cloth. They adsorb
the dye particles which, otherwise, do not stick to the cloth.

4. In dehumidizers:Silica gel is commonly employed to adsorb humidity or moisture


from air. This is necessary for the storage of delicate instruments which might otherwise
be damaged by contact with most air.

5. Heterogeneous catalysis:The use of the metal catalysts such as Fe, Ni, Pd, etc., in
the manufacturing processes such as Contact process, Haber process and the
hydrogenation oils, is based upon the phenomenon of adsorption.

6. In ion-exchange resins:The organic polymer containing groups like –COOH, –


SO3H, –NH2 etc., possess the property of selective adsorption of ions from solutions.
These are quite useful in the softening of water and also in the separation of the elements
of the Lanthanide series (also called rare earths).

7. In Chromatography:The different chromatographic techniques such as adsorption,


paper or column chromatography which are used for the purification and the separation of
the substances available in small amounts, are based upon the theory of selective
adsorption.

8. In qualitative analysis:Certain qualitative tests such as the lake test for the
confirmation of Al3+ ions are based upon adsorption, i.e., Al(OH)3 has the capacity to
adsorb the colour of the blue litmus from the solution.

9. In adsorption indicators: In many titrations involving precipitating reactions, dyes


such as eosin and flourescein are used as adsorption indicators. This is based upon the
fact that these are more adsorbed by the precipitates than by the solution.

The Colloidal State


Thomas Grahma in 1862 coined the term colloids (Greek Kollo– glue, iods– like) to distinguish
some amorphous glue-like substances from crystalloids which crystallize easily. He distinguished
these two substances on the basis of their diffusion through parchment membrane.

1. Crystalloids: The one having faster rate of diffusion.


2. Colloids: The one having slower rate of diffusion.

However, this classification has now historical importance and was rejected because some
substances diffuse rapidly in water but slowly in alcohol medium (e.g. NaCl), whereas some
substances diffuse slowly in water but rapidly in alcohol medium (.e.g soaps). Therefore, a new
term colloidal state was preferred.

It is therefore convenient to examine the colloidal state and not the colloids as such. Though
Graham’s concept of colloids is not fully valid, the term colloids has, however, been retained.

Characteristics of Colloids

1. It is a particular state and not a class of compounds.

2. It is the state in which rate of diffusion through parchment membrane is slow.

3. It is heterogeneous state, i.e., number of phases ≥ 2, i.e, each particle is contained within
its own boundaries which separate it form dispersion medium.

4. Thus a colloidal state must have at least two phases. The one phase dispersed in other is
known as dispersed phase (D.P.) or internal phase whereas the other in which dispersions
are made is known as dispersion medium (D.M.) or external phase.

5. It is an intermediate state in between true solution state and suspension state, i.e., particle
size of colloidal state (of dispersed phase) lies in between true solution state and
suspension state.

True solution Colloidal state Suspension

Solution + Solvent D.P. + D.M. Precipitate + medium

Size: Solute < 1 × 10–7 cm DP = 10–5 to 10–7 cm Precipitate = 0.2 × 10–4 cm

6. Particles in a colloidal state are not detectable even under microscope; however, their
images can be noticed in ultra microscope.

7. Colloidal particles do not settle down under the force of gravity even in keeping for long.

8. Colloidal particles always carry charge, +ve or –ve on dispersed phase particles.

Distinction in true solutions, colloids and suspensions

A colloidal solution differs from true solutions and coarse suspensions due to the size of solute
particles. Usually the size of the solute in true solution is of the order of atomic/molecular size
10–7 cm) whereas in coarse dispersion or suspension the particle size exceeds 10–4 cm. the
colloidal size lies between the two extremes of true solutions and coarse dispersions. In table are
shown some distinctions. Though a clear cut boundary on the basis of particle size is not
established, colloidal particles are often considered in the range 10–5 – 10–7 cm. This range is not
microscope or an electron microscope. However, emulsions, which are also considered in the
category of colloids and often have particle size towards longer side of the colloidal range, ~10–4
cm are visible under the ordinary microscope.
Coarse suspension Colloidal system True solution
Particle s shown in solid circles are those of continuous medium
(i.e. molecules of the order of angstrom i.e. 10–8 cm)
Solute particles (molecules) in true solution of the size of the
continuous medium colloidal particles range (10–510–7) whereas coarse
suspension have even bigger sized particles.
Figure 7: The colloidal dispersions

Alyophobic colloid is always heterogeneous. It consists of atleast two phases immiscible with
each other. Thus a colloidal dispersion made by two different immiscible substances contains one
phase in the form of dispersed particle called dispersed phase which remain present in a
continuous medium of the other substance called dispersion medium. Some general differences
in true solution, colloidal solution and coarse suspension are given in table.

Table 2: Distinguishing features in suspension, colloids and true solution


Property Suspension Colloid Solution
Particle size > 10 cm or 10 Å or 100 mµ
–5 3 –7 –5
10 to 10 cm or 10Å to –7
<10 cm or 10Å or 1 mµ
103Å or 1 mµ to 100 mµ
Visibility Visible with naked eye Images are visible under Not visible with any of the
ultra microscope optical means
Separation
a) with filter paper Possible Not possible Not possible
b) with membranes Possible Possible Not possible
Diffusion Does not diffuse Diffuses very slowly Diffuses readily
Setting or sedimentation Settles under influence of Settle under centrifuge Does not settle
gravity
Nature Heterogeneous Heterogeneous Heterogeneous
Appearance Opaque Turbid Clear

Classification of Colloidal State

1. Classification based on the nature of dispersion medium:If dispersion


medium is

– air then aerosol


– water then hydrosol or aquosol
– benzene then benzosol
– alcohol then alcosol

2. Classification based on the nature of charge

If dispersed phase particles carry +ve charge; it is +ve sol


If dispersed phase particles carry –ve charge; it is –ve sol
3. Classification based on the physical state of dispersed phase and
dispersion medium: Considering the three physical states of aggregation of all matter
as solids (S), Liquid (L) and Gas (G), eight different categories exists taking two
immiscible substances as dispersed phase and dispersion medium as described in table.

Table 3: Some examples of different class of Lyophobic Colloidal Systems


Internal phase or External phase or Colloidal Example
dispersed phase dispersion medium name
Alloys, ruby glass, Gems
Solid or Precious stones,
Solid
Solution Marbles, Optical and
vision glasses.
Solid
Muddy water, gold sol,
Liquid Sol
paints, cell fluids
Smoke, particulate clouds,
Gas Aerosols
dust
Alloys, ruby glass, Gems
or Precious stones,
Solid Gel
Marbles, Optical and
vision glasses.
Liquid
Muddy water, gold sol,
Liquid Emulsion
paints, cell fluids
Smoke, particulate clouds,
Gas Aerosols
dust
Purnice stone, polystyrene
Solid Solid form
foam, foamed rubber.
Foam or Froth, soap suds, whipped
Gas Liquid
froth cream
Homogenous
Gas Do not exits as colloids
system

4. Classification based on affinity of dispersed phase for dispersion


medium:The substance that remains present as the dispersed phase does not have any
affinity towards the solvent (dispersion medium) and these colloids are thus called
Lyophobic colloids. A lyophobic colloid thus needs some special methods of preparation.
This can be done in two ways. In the first case, the substance that forms the dispersed
phase, is finely divided (dispersion method) or is condensed by forming aggregates of
atoms/molecules (condensation method). These two methods of preparation of lyophobic
colloids has been dealt later in this chapter along with their characteristic features.

Lyophilic and Lyophobic Colloids: Apart from lyophobic colloids, some substances,
when simply, placed in the dispersion medium, dissolve and form colloidal solutions. The two
substances are not immiscible with each other as in case of lyophobic colloids. Such colloids
solutions are called Lyophiliccolllid due to the affinity of the solute with the solvent. How then
the colloidal properties develop in such systems? As has been made clear that colloidal solutions
are typical in that they contain particles in the range 10–5 to 10–7 cm.
Lyophilic Colloids

1. Association Colloids:Soap and detergent when dissolved in water show the


aggregation and form colloidal particles called micelles.

CH2 CH2 CH2 CH2 O


H3C CH2 CH2 CH2 C Na
Nonpolar tail
(hydrophotic) O
Polar tail (hydrophilic)

10-5-10-7cm
(Collidal range)

2. Macromolecular Colloids:Large polymer molecular when dissolved in a liquid


molecularly dissolve. Very high molecular mass of polymer gives colloidal dimension to
its molecule.

10-5-10-7cm

When a polymer (a high molecular weight substance) is placed in a solvent, it dissolves at


molecular level but because of high molecular mass (1000 times or even more for example,
compared to low molecular weight materials), the polymer molecule itself becomes of the size
comparable to the colloidal range. Such lyophilic colloids, which are molecular solutions of
polymers are called macromolecular colloids. Apart from this class of lyophilic colloids, the
solution of soaps and detergents in water from lyophilic colloidal dispersions. When placed in
water, a soap or detergent dissolves at molecular level and forms true solutions. However, if the
concentration of soap is high, hundred of molecules of soap aggregate together by physical forces
and form a colloidal entity called Micelle.

The formation and properties of micelles will be dealt later in this unit. Soaps and detergents for
their aggregation behaviour that leads to colloidal solutions are also known as Association
colloids. Thus colloidal solutions of soap and detergents and those of high mol mass polymeric
materials (e.g., starch, gelatin, gums) in water are collectively known as hydrophilic solutions.
These are shown in figure.

No special method is employed in making of a lyophilic dispersion (simple dissolution is


enough). For example, gelatin, agar-agar, starch etc. when mixed with hot water get converted
into sols. In principle any molecular solution of a high polymer in any solvent can be called as a
sol. Lyophilic sols (of association colloids) can be made by simply dissolving soaps and
detergents in above certain concentration. These solutions are thermodynamically stable and
cannot be coagulated (to create dispersed phase material from dispersion medium i.e.,
precipitation or breaking up of solid).
In table, the two types of sols are distinguished from one another.

Lyophilic and Lyophobic Sols


Property Lyophilic Sols Lyophobic Sols
Nature Dispersed phase has more Dispersed phase has less
affinity for dispersion medium affinity for dispersion
medium.
Preparation Need no special methods for Simple mixing does not
preparation. Adding of the produce sols. Special
material in solvent gives the methods (dispersion or
sols. For example adding starch condensation) are necessary.
in water gives starch sol
Concentration Sols have higher concentration Sols have lower
of dispersed phase. concentration of dispersed
phase.
Stability Thermodynamically more Thermodynamically less
stable systems stable systems. Coagulate
with time.
Solvation Dispersed phase particles are Dispersed phase particles are
highly solvated not solvated.
Viscosity Viscosity of sols are often Almost same as that of
higher than the viscosity of dispersion medium.
dispersion medium.
Surface tension Surface tensions of sols are Almost same as that of
much lesser than the surface dispersion medium.
tensions of dispersion medium
Reversibility Reversible with temperature. Irreversible.
Charge The charge on dispersed phase Independent of pH of
particles depends upon pH of medium.
medium.
Tyndall effect Less scattering of light. More scattering of light.
Coagulation by electrolytes Very high concentration of salt A small amount of salt is
is required for phase separation. required for their coagulation.

The lyophobic sols consist the colloidal particles in the form of aggregates of small molecules or
atoms (gold sol, sulphur sol, As2S3 sol etc. are some examples) which are held together by van
der Waals forces. These are thus also called as multimolecular colloids.

The surface area of colloidal particles in a colloidal solution is very large. This can be nicely
illustrated considering one cube of 1 cm dimension. Its total surface area will be 6 cm2. Now if
this cube is cut into several cubes of 1 mm length, the total number of cubes will be 1000 with
each cube of total surface area = 6 mm2 = 0.06 cm2. For 1000 cubes this area will be
0.06 × 1000 cm2 = 60 cm2. Further subdivision say to cubes of 1 µm size will give a surface area
of 6000 cm2 and to 1 mm size will give a surface area as high as 6 × 107 cm2. It is due to the large
surface area of colloidal size particle, that adsorption on the particle is of great importance. It
fact, colloid chemistry and surface chemistry are highly interrelated fields.
Preparation of Colloids: The two different techniques that can be employed to obtain a
lyophobic sol are condensation methods and dispersion methods. In condensation method, as the
name suggests, the atoms and molecules of the substance condense (aggregate) leading to the
particles of colloidal dimension. The condensation can be brought about by both physical and
chemical means.

The Physical methods are illustrated below:

i) By exchange of solvent:A true solution of a solute in a solvent when mixed with


another liquid (which is miscible with the solvent but does not dissolve solute in it) then a
colloidal system is produced. For example, sulphur is molecularly dissolved in alcohol; an
addition of excess amount of water to this solution produces colloidal solution of sulphur
in water.

ii) By excessive cooling:Cooling sometimes produces colloidal dispersions. An example


of the method is to produce colloidal solution of ice in organic solvents like ether,
chloroform etc., by freezing a mixture of solvent and water.

Using Chemical Methods, a sol can be prepared by hydrolysis, reduction, oxidation, double
decomposition etc.

i) By hydrolysis:Addition of drop by drop of an aqueous solution of FeCl3 in boiling


water.

FeCl3 + 3H 2O 
→ Fe(OH)3 + 3HCl
sol

ii) By reduction: gold sol can be prepared on reducing a solution of gold (III) chloride
with formaldehyde and tannic sol.

2AuCl3 + 3HCHO + 3H 2O 
→ 2Au + 3HCOOH + 6HCl
sol

Gold sol can be prepared by reducing AuCl3(aq) by hydrazine or SnCl2.

4AuCl3 + 3NH 2 NH 2 
→ 4Au + 3N 2 + 12HCl
sol

2AuCl3 + 3SnCl2 
→ 2Au + SnCl4
sol

iii) Oxidation:Sulphur sols are obtained when H2S gas is bubbled through the solution of an
oxidisng agent like Br2, HNO3, Mn2+ etc. where H2S is oxidized to elemental sulphur.

H 2S + Br2 
→ 2HBr + S
sol

H 2S + 2HNO3 
→ 2H 2O + 2NO 2 + S
sol

Therefore during 2nd group analysis of mixture, by passing H2S in cold solution of
mixture sometimes (if it contains an oxidant) leads to the formation of white, grey
turbidity of sulphur sols instead of precipitates of 2nd group sulphides.
A colloidal solution of sulphur can also be obtained by adding dil. HCl to sodium
thiosulphate.

Na 2S2 O3 + 2HCl 
→ 2NaCl + SO 2 + H 2 O + S
sol

iv) Double Decomposition: This method is often used to prepare various colloidal
solutions. Sulphide sols are often obtained by this method. For example, on passage of
H2S through cold aqueous solutions of AsCl3, CdCl2, Hg(CN)2, their sulphide sols are
formed.
2AsCl3 + 3H 2S  → As 2S3 + 6HCl
sol

CdCl2 + H 2S 
→ CdS+ 2HCl
sol

Hg(CN) 2 + H 2S 
→ HgS+ 2HCN
sol

Also, drop wise addition of FeCl3 solution to solution of sodium ferrocyanide gives
Prussian blue sol.

4FeCl3 + 3Na 4 [Fe(CN) 6 ] 


→ Fe 4 [Fe(CN) 4 ] + 12NaCl

Silver chloride sols, Barium sulphate sols are formed by allowingthe following reactants to react
in cold, dilute conditions.
AgNO3 + NaCl  → AgCl+ NaNO3
sol

BaCl2 + H 2SO4 
→ BaSO 4 + 2HCl
sol
The dispersion methods employed for making lyophobic sols involve the following techniques.

i) By Grinding:The coarse particles of Suspension


Driving belt
dispersed phase in presence of Hollow shaft

dispersion medium are grinded into


smaller colloidal size particles in Discharge Discharge

grinding machines such as mixie or


colloidal mill (figure). These fine
particles can then be stabilized by an Metal disc

stabilizer, for example, gum, tannin etc.


(Dispersion method)

ii) By Electrical Method:This is High voltage source

a simple method (Figure) for Gold electrodes


preparing metal sols (e.g., gold
sol) and involves an electric are
between the two metal rods
Ice bath
partly dipped into the water as
dispersion medium. The high Water

electric tears the particles from H2O + little


rod (or wires) which are of KOH

colloidal size and remain


dispersed in dispersion medium.
The heat generated during the process is removed by putting system under ice cold water.
Little amount of KOH in water prevents growth of sol particles and stabilizes the
dispersion. Purple of Cassuis (Au sol in water) sol is obtained by this method.

iii) Peptization

1. The phenomenon of converting fresh precipitate into colloidal state by the action of
solute or solvent is known as peptization. The solute or solvent used are known as
peptizing agent.

2. For example
a) Freshly precipitated Fe(OH)3 turns into in colloidal state by the action of FeCl3.


FeCl3

Small colloid particles of Fe(OH)3 stabilized


Freshly precipitated ferric hydroxide
by preferential adsorption of Fe+3

Peptization – A method for the preparation of a sol


b) Freshly precipitated stannic acid can be peptized by NH3(l) or HClaq.
c) Freshly precipitated HgS, CdS and NiS can be peptized by H2S.
d) Peptization of albumin in water of nitrocellulose in acetone or ethyl alcohol to
form collodion.

3. The process of peptization thus involves the adsorption of suitable ions (supplied by
the electrolyte added – particularly a common ion) and electrically charged particles
then split from the precipitate as colloidal particles. A coarse suspension of silver
chloride on shaking with electrolytes produces AgCl sol. Shaking of AgCl suspension
with AgNO3 solution produces positively charged AgCl sol due to adsorption of Ag+
ions whereas shaking of AgCl suspension with HCl produces negatively charged
AgCl sol due to adsorption of Cl– ions.

Purification of Sols:

Sols so obtained are contaminated with two types of impurities and need purification.

a) Insoluble impurities:These are removed by simple filtration of impure sols. Impurities


are retained on filter paper and sols are filtered.

b) Soluble Impurities:The soluble impurities either of molecular nature or of ionic nature


in colloidal solutions need special methods for their removal from sols.

Following techniques are used for this purpose.

Dialysis:The separation of soluble Dialysis bag Solvent


impurities from a colloidal solution on
Colloidal particle
the basis of their different rates of
Small molecules
diffusion through parchment membrane (or ions)
is known as dialysis. Solvent impurities

Figure: (a)
This can be done by using a bag made of
a semipermeable bag. It has small pores Dialysis bag Solvent
which allows the small molecular / ionic Colloidal particle

species to pass through but not the Anode (+) Cathode (–)
Small molecules
colloidal particles which are larger in size (or ions)
Solvent
as compared to the membrane pores. impurities

Thus, a colloidal solution is filtered in the Figure: (b)


bag (figure a) which is tied on the mouth
and suspended in the liquid (the
dispersion medium). Slow diffusion of
molecules/ions from colloidal solution to
the outer medium purifies the sol.
In case of ionic nature of impurities, dialysis can be speeded up by taking an advantage of
electricity and the process is known as electrodialysis (figure b).

Properties of Colloids: Due to their small size, these colloidal solutions remain stable over
a long period of time as they do not settle down under earth’s gravity (sedimentation rate is very
slow). At the same time most lyophobic colloids are stable due to the surface charge or a layer of
a protective material on to the surface of colloidal particles. This surface modification does not
allow the isolated particles to come close to each other, colloid and form bigger particles.
However, as the systems are associated with large surface area and consequently large surface
free energy, these systems have to break one day. Thus, lyophobic colloids are
thermodynamically unstable systems.

Large surface area provides a tremendous adsorption capacity to the colloidal particles. This
property is utilized, for example, to remove poisonous gases by use of colloidal charcoal in gas
masks and removal of effluents like dyes etc. from waste water.

Following are some characteristic properties of colloidal solutions.

1. Physical Properties
a) Heterogeneity:Sols are heterogeneous in nature.

b) Diffusibility:Sol particles have slow rate of diffusion through parchment


membrane.

c) Filtrability:sol particles can pass through ordinary filter papers.

d) Non settling nature:Sol particles do not settle down due to gravity.

e) Viscosity and surface tension:Viscosity and surface tensions of sols are almost
similar to those of pure solvent in case of lyophobic colloids. In lyophilic colloidal
solutions, viscosity is higher than the solvent and surface tension being lower than the
solvent used as dispersed medium for sols.

f) Visibility and Size of the Particles

i) It is not possible even by the help of a most powerful microscope to see colloidal
particles because clear image formation of a particle smaller in size than the
wavelength of light used is not possible. However, zones, of scattered light can be
seen by a microscope and some times with naked eye also.

ii) The upper limit of colloidal particle size has been obtained as the lower limit of
microscopic visibility. It is not possible to see any particle whose diameter is less
than half of the wavelength of visible light, i.e., (1/2) × 4000Å i.e. 2000Å since
sol particles are invisible even under microscope and thus their upper limit is
2000Å or 2 × 10–5 cm or 0.2 µ or 200mµ. The lower limit has been taken to be
10Å since particles lesser in size than10Å are assumed to be true solution
particles. Thus size of sol particles lies in between 10Å to 2000Å.

g) Surface area and adsorption:The surface area of the particles in colloidal state is
appreciably larger than compared with an equal mass of the matter in coarse grained
size. It is due to the larger surface area, sol particles shown high tendency for
adsorption. The property has got application in medicinal chemistry where drugs are
sold in from of sols.

2. Colligative Properties:Like true solutions, colloidal solutions also exhibit colligative


properties such as osmotic pressure, elevation in boiling point, depression in freezing
point, lowering in vapour pressure. Since colligative properties of a solution depends
upon the number of particles present in solution as well as on their weight. Since in
colloidal state, there exists aggregates of thousands of molecules of high molecular
weight and thus number of particles being relatively lower than true solutions and
therefore the colligative properties are lesser than those observed for true solutions
containing the same amount.

3. Optical Properties: Tyndal effect:All colloidal solutions are capable of scattering


light or iopalescence.
Not
When a beam of converging Visible visible
rays falls in colloidal Light
solution, scattering of light source

by sol particles in all the Light beam

directions, gives rise to a


bright glowing cone when
looked at it sideways. This is Colloidal Solution True solution
known as Tyndall effect. Figure: Light scattering by colloid dispersions (Tyndal effect)
The sol particles absorb light, become self luminous and then scatter light in all possible
directions.

The scattering of light depends upon the


i) Wavelength of light used:Scattering ∝ (1/λ4); That is why scattering of blue light
is maximum.
ii) Size of particles:Larger size in dispersed phase particles show more scattering.
That is why lyophobic sols shows more scattering.
iii) Difference in refractive indices of dispersed phase and dispersion
medium, (i.e. ∆(µP ~ µM):Larger is ∆µ, more is scattering.
Some examples of Tyndal effect are:
i) Blue colour of sky and sea water.
ii) Visibility of tails of comets.
iii) Twinkling of stars.
iv) Visibility of sharp ray of sunlight entering through a slit in dark room.
v) Visibility of projector path and circus light.

True solutions do not show Tyndall effect because the size of molecules or ions here is
too small to cause scattering.

4. Mechanial Properties: Brownian


motion:English Botanist Robert Brown noticed
the irregular or chaotic motion of (pollen grains)
particles suspended in water.This was later on
named as Brownian motion. Later on Zsigmondy,
after the discovery of ultrmicroscope (based on
Tyndal effect) noticed the similar irregular motion
of the images of sol particles (figure) and
concluded that colloidal particles move much more
vigorously than small microscope particles Figure: Brownian movement the zig-sag
motion of a colloidal particle.

Following are the important characteristic conclusion regarding Brownian motion of


particles.
i) Brownian motion is observed upto a particle size of 0.5 microns.
ii) The smaller the particle, the more rapid is the movement.
iii)The movement is independent of λof light used and time of illumination.
iv) The movement does not change with time and exist till last.
v) The movement depends upon temperature; the intensity of the movement increases
with increasing temperature.
vi) The movement is due to the bombardment of solvent and the movement increases
with increasing temperature.

Applications of Brownian motion are


i) It offers an explanation for the stability of colloidal solutions.
ii) It offers an experimental set up to determine Avogadro’s number.
iii) It offers an explanation for kinetic theory of matter.

5. Electrical Properties:
Electrophoresis:Colloidal particles move Electrode

towards the oppositely charged electrode when


an external electrical field is applied. This Coagulated sol
particles As2D3 sol
phenomenon is called electrophoriesis. A set (negatively
charged)
up demonstrating electrophoresis consists of
U-tube filled with colloidal solution and
attached with electrodes as shown in figure.
On applying direct current (dc), colloidal
particles move and coagulate. Figure: A set up for electrophoresis
Charge on sol particles is a fundamental factor for their existence. Generally dispersed
particles with less charge coagulate. It is the charge on sol particles which keeps them
apart and prevent them from coming closer to each other to coagulate. The charge may be
+ve or –ve. For example, Metal hydroxide sols are –ve sol, smoke is –vely charged.

The stability and instability of colloids and coagulation: The stability of a


colloidal dispersion means it to remain in unaltered condition over a certain period of lime. The
instability means the breaking down of a colloidal dispersion into two phases.

i) Spontaneous breaking up of a colloidal solution, i.e., the separation of dispersed phase


from dispersion medium or destabilization of sol is known as ageing.

ii) Destabilization of sols may also be made by artificial means and the phenomenon is
known as coagulation.

The stability of a colloidal dispersion is usually achieved by using finer particles, inducing
charge on the particles or by using the dispersion of high viscosity. Stable lyophobic dispersions
can also be made by the coating of some polymers like starch, gelatin, gum etc. (These
substances are called protective colloids and their efficiency to stabilize a lyophobic dispersion is
measured in terms of gold number).

Conversely, the instability or the coagulation of sol can be achieved by adding a substance
(called coagulant) which will remove the protective layer or will neutralize the charge from the
surface of colloidal particles.

Following are some general methods used for coagulation of sols.


a) General methods: 1. By heating: Coagulation of butter
2. By cooling: Coagulation of milk, fat floats over
3. By exposure to light
4. By mechanical agitation
5. By persistent dialysis
b) Cataphoresis:Sol particles during migration under cataphoresis towards opposite
electrodes get adhered and lose their charge.

c) Mutual coagulation:A deep red +ve sol of Fe(OH)3, on mixing with bright yellow –ve
sol of As2S3 shows mutual coagulation of both leading to a colouress solution with
precipitate settled down due to neutrlalization of charges by each other.

The coagulation of sols by electrolytes has been dealt in terms of Hardy-Schulze rule. According
to this rule:
1. One of the ions furnished by an electrolyte, carrying charge opposite to that of sol
particles is responsible for coagulation of sols and is known as effective ion or
counter ion.
2. The greater is the valency of the counter-ions, the greater will be its coagulating power.
For example, in coagulating, a negatively charged sol (e.g., As2S3 sol) the order of
efficiency of coagulating ions will be Th4+ > Fe3+> Ca2+> Na+. Similarly, for a positively
charged sol say Fe(OH)3 sol, the order will be [Fe(CN)6]4–> [Fe(CN)6]3–> SO42–>Cl–.
3. More is the coagulating power, lesser is coagulation or flocculation value (i.e., the
minimum amount of electrolyte required to coagulate a definite amount of (sol).

1
Usually, the flocculating value of these ions are quantitatively described as where x
(x)6
is the valency of the coagulating ion. For example, for mono-, di-, tri- and tetra-valent
ions, the ratio in the flocculation value will be

1 1 1 1
6
: 6: 6: 6 or 100 :1.56 : 0.137 : 0.025
(1) (2) (3) (4)

Thus if 100g of a monovalent ion is required to coagulate a sol, only 0.025g of a


tetravalent ion will be necessary. Most natural colloids e.g. blood, milk, aerosol etc., bear
negative charge and to our common experience alum (Al3+) is used to remove the
suspended impurities to clarify was or to stop bleeding by clotting (or coagulating )
blood.

Coagulation values of cations and anions are reported in table.


For negatively charged sols For positively charged sols
Coagulation Coagulation
Electrolyte Positive ion Electrolyte Negative ion
value value
3+ –
FeCl3 Fe 0.136 KCl Cl 103
CaCl2 Ca2+ 0.649 KNO3 NO 3− 131
BaCl2 Ba2+ 0.691 K2SO4 SO −4 0.22
KCl K+ 49.5 K2CrO4 CrO 24 − 0.33
+
NaCl Na 51.0 K4[Fe(CN)6] [Fe(CN)6]4- 0.06

Addition of electrolyte to sol particles, neutralizes their charge and thus electrokinetic potential
approaches to zero. At this point sol particles do not carry charge and are either coagulated or just
to coagulate; however, they become inert towards cataphoresis. This condition is known as
isoelectric point, i.e., the H+ ion concentration at which the colloidal particles do not carry charge
and possess minimum stability.

Following are some applications of coagulation:


a) Coagulation of latex (sol) gives rubber.
b) Removal of carbon from smoke (–ve of carbon in air) is down on the basis of charge
neutralization using Cottrell’s precipitators.
c) Sewage water disposal is done by using coagulation technique. Dirt, dust particles
suspended in water are coagulated by adding salts or polymers.
d) Alum (a very good coagulating agent) is used to purify impure water having clay particles
suspended in it.
e) River water contaminated with various sol particles in it, on meeting with ocean shows
their coagulation by salts present sea water and forms delta.
Protection of Colloids
i) Lyophobic sols are less stable than lyophilic sols. However, their stability may be
increased on addition of lyophilic sols. This phenomenon of stabilizing lyophobic sols by
the addition of lyophilic colloids is known as protection. The lyophilic colloids used for
this purpose are known as protective colloids.
ii) The protective colloids form a thin layer around the dispersed phase of lyophobic colloids
and thus prevent them from coming close to each other and stabilize them.
iii) The protective character of various lyophilic substances is expressed in terms of gold
number.
iv) Gold number of lyophilic is defined as the minimum amount of lyophilic colloid in
milligrams which prevents the coagulation of 10 ml gold sol by 1 ml of 10% NaCl
solution.
v) More is the gold number, lesser is the protective power of lyophilic colloid.
Substance Gold Number
Gelatin 0.005 – 0.01
Haemoglobin 0.03 – 0.07
Gum Arabic 0.15 – 0.25
Sodium oleate 0.04 – 1
Starch 15 – 25
vi) Gelatin, protein, gums and other macromolecules etc. are popoular protective agents.
vii) A hydrophilic substance does not protect all hydrophobic colloids and protecting power
varies for different lyophobic sols. Therefore, another term Congo robin number was
introduced to express protective power of lyophilics. Congo robin number is defined as
the amount of protective colloid in milligrams which prevents colour change in 100 ml of
0.01% Congo robin dye solution to which 0.16 equivalent of KCL are added.
viii) It has been found that protective action of hydrophilics is altogether different for gold and
Congo robin sols.

Lyophilic colloids on the other hand are stable dispersions. In principle, these would not
coagulate but the dispersed phase can be precipitated from the dispersion medium by the addition
of large quantities of salts (‘salting out’ action of electrolytes) or by adding a liquid which is
miscible with water (dispersion medium). Both these methods work by decreasing the interaction
of dispersed phase with dispersion medium.
i) Addition of electrolyte: Salting out effect:When a large amount of an electrolyte
is added to a lyophilic sol, coagulation takes place. The excess electrolyte removes
solvent layers because of solvation of the added electrolyte and thus makes available less
solvent for solvation of colloidal particles. This leads to unstability and ultimate
coagulation of particles.
ii) Addition of a liquid in which solvent is solute: When alcohol or acetone is added
to an aqueous hydrophilic sol, water being miscible with alcohol results in the removal of
solvent layers and hence precipitation of sol.
Emulsions
Emulsions are colloidal dispersion of one liquid into another liquid (the two liquids should
beimmiscible). Normally one of the two liquids is water.

Emulsions are broadly classified into two types:

i) Oil-in water (or o/W)


ii) Water-in-oil (or w/o)

An oil-in-water emulsion consists of droplets of oil dispersed in water. In a water-in-oil


emulsions water is present as dispersed phase with oil as the dispersion (or continuous) medium.
Which type of emulsion will form on mixing oil and water along with an emulsifying agent will
depend on the composition of the oil and water (usually the liquid with higher percentage
becomes dispersion medium), nature of emulsifying agent (usually water soluble substance
stabilize o/w emulsion whereas oil soluble substance favours the formation of w/o emulsions).
The two types of emulsions are shown in figure. Also a comparison of o/w and w/o emulsion is
shown in table.

Water
OIL
Oil

WATER

Oil and water Oil in water Water in oil


do not mix emulsion emulsion
Figure: Use of emulsions e.g. soap and detergent form emulsions

Preparation of Emulsions: The process of making in emulsion is known as emulsification.


Emulsions are obtained by vigorously mixing both the immiscible liquids. However, this gives an
unstable emulsion. The dispersed drops at once come together and form separate layers. To
stabilizes an emulsion, the addition of a small quantity of a third substance known as emulsifying
agent or emulsifier is essential. Soaps and detergents and polymers are most frequently used as
emulsifiers. They coat the dispersed droplets of an emulsion and check them from coming
together.

Comparison of the properties of two types of emulsions


Characteristics Water-in-oil emulsion Oil-in-water emulsion
Dispersed phase Water Oil
Dispersion medium Oil Water
Appearance Oily, opaque and translucent Watery, opaque and
(permits light to pass through translucent.
but diffusing it so that objects
on the other side are not
clearly visible).
Electrical conductivity Very low Appreciable
Characteristics Water-in-oil emulsion Oil-in-water emulsion
Viscosity More than water Little more than water.
Dilution test More oil added is soluble but More water added is soluble
not water. but no oil.
Spreading test Spreads easily on oil surface Spreads easily on water
surface.
Dye-test with oil soluble dye Dye is soluble giving a bright Dye is practically insoluble,
e.g., methylene blue. colour. Butter, cold creams, coloured droplets are seen.
Examples benzene water, aniline-water Milk, cream, face creams.
etc.

Emulsions are prepared by vigorous shaking of the relevant liquids or by subjecting a coarse
mixture to ultrasonic vibrations. Emulsions, like other colloidal systems are unstable and are
stabilized by the addition of a third component known as emulsifying agent or emulgents or
emulsifiers. In the absence of an emulsifying agent, the dispersed droplets coalesce together and
the two liquids are separated into separate layers. Commonly used emulsifiers a surfactants,
polymers and metal oxides and hydroxides in finely divided states. The function of an
emulsifying agent is simply to absorb onto the oil-water interface and impart stability.

Identification of Emulsion Type: Two common and routinely used methods for quickly
identifying the emulsion type are:

i) Dilution test:An oil-in-water emulsion (where water is continuous phase) can be


diluted by water. On adding water to water-in-oil emulsion, will form a separate layer.
For example, milk which is an oil-in-water emulsion can be diluted by water but butter
which is water-in-oil emulsion cannot be diluted by water.

ii) Dye test:A water soluble dye will give uniform coloured background when added to oil-
in-water emulsion. Similarly, an oil soluble in dye will give colour background on adding
to water-in-oil emulsion.

Emulsions exhibit all those properties as shown by lyophobic sols. These are also
thermodynamically unstable and can be coagulated by adding electrolytes. The only difference in
the properties of sols and emulsions can be of the particle size. Emulsions are usually in the
larger limit of colloidal range (10–5 – 10–7 cm) and are often of the size of the order of a micro
(=10–4 cm). The large size of emulsion droplets make emulsion droplets opaque or translucent.

Emulsions have found wide applications in day to day life in cosmetics (e.g., creams, hand
lotions, cleansing milk are all emulsions), pharmaceuticals (e.g., ointments), agricultural, textile
surface by soaps and detergents principally involves emulsification. Digestion of fats in the
intestine is aided by bile salts which act as emulsifying agent to form fat in water emulsions.

Emulsions are not always desirable. Sometimes unwanted stable emulsions are formed by nature
that need to be broken, the breaking of an emulsion is called demulsification. For example, the
crude petroleum obtained by digging wells in oil fields is always associated with some water
which is in the form of water-in-oil emulsion. The crude oil is to be freed from water before
sending it to the refineries for fractionation. Here demulsification plays an important role. An
emulsion can be demulsified by physical methods like freezing, boiling or centrifugation. For
example, centrifugation provides milk cream and remaining skimmed milk contains very little
amount of fat. The chemical methods involve the use of substance (demulsifiers) which destroy
the emulsifying agent from the oil-water interface which is responsible for the stability.
Demulsification means the breaking of an emulsion into aqueous and nonaqueous phases.

Association Colloids
These colloids are better known as surface active agents (or surfactants). They do show surface
activity, i.e., the property to lower the surface tension of liquids or the tendency to increase
surface area.

A surface active substance has in its


molecule distinct hydrophobic and
hydrophilic part. When dissolved in
water, their aqueous solution contains
isolated molecules or ions (if the
surface active agent is an electrolyte Soap molecules
like a soap which is RCOO– Na+) upto Soap micelle
a certain concentration. However, i) Micelles form when sopa conc. exceeds CMC

above a certain concentration these ii) Micelles are colloidal size entities with 100-200 soap moelcules

molecules or dissociated ions iii) Micelles may be spherical or cylinderical in shape

aggregate by themselves such that the Figure: The miscelles


nonpolar tails (hydrophobic part) of
the surface active agent does not have any interaction with water. Such a thermodynamically
favourable situation results in the formation of particles of colloidal dimensions, usually
spherical in shape and are termed as Micelles (figure). Micelles are the aggregates consisting of
several tens or few hundreds of molecules or ions of a surface active substance when it is
dissolved in a solvent usually water. The soap concentration at which the micelles first appear is
called is called the critical micellar concentration (CMC) and the number of molecules that
constitute one micelle is called aggregation number.

The formation of micelles is a result of the hydrophobic interaction which allows the aggregation
of nonpolar (hydrophobic) tails of the surfactant. In this way the polar head groups only remain
in contact with water.

For example, a soap sodium laurate (C11H23COO–Na+) or detergents, like sodium lauryl sulphate
(C12H25SO4–Na+) or sodium salt of linear alkyl benzene sulphonate (LABS) will dissolve in water
and will form ions, say laurate ion and sodium ion from sodium laurate. Above about 100 mM
sodium laurate concentration, the micelles with an aggregation number of about 100 will form.
These spherical aggregates formed from several laurate ions are of colloidal size and will provide
the soap solutions all the properties of colloids like light scattering, electrophoresis, viscosity etc.

Micelles have the core usually made of a non-polar portion of the soap. Thus these are capable of
solubilizing substances otherwise insoluble in water. For example, fat soluble vitamins like A,E,
E, K and stored drugs can be made soluble in aqueous solutions of micelles.
Applications of Colloids
Colloidal dispersions play a very significant role is natural processes in our daily life and in
industries. Some of the important applications of colloids are in:

Food:Most of our food preparations are colloidal in nature. Milk, butter and cream are
example of emulsions, Jam, jellies, ice-creams, salad dressing, bread etc., are all
examples of food colloids.

Medicines:Most drugs (insoluble in water) are administered as colloidal dispersions


namely suspensions, emulsions. In the colloidal form the drug is easily absorbed by body
tissues and is therefore more effective. Most skin ointments are emulsions.

Detergency:Soaps and detergents by themselves are lyophilic colloids and therefore


the entire detergent industry involves colloids.

Water treatment:Suspended/colloidal particles of undesirable muddy mass need be


removed before making it fir for drinking. The water cleansing process involves the
coagulation of the suspended material by alum or high mol mass polyelectrolytes in water
treatment plants.

Precipitation of Smokes: Cottrell smoke High voltage electrode


(30000 volts or more)s
precipitater is a device installed in the chimney of Gases free from
an industrial plant emitting gaseous waste as carbon particles

colloidal system (serosols) with unburnt solid


particles as dispersed phase. The precipitator has
(Figure) two metal discs charged to a very high
potential. Such a device neutralizes the charged
suspended unburnt solid matter which precipitates
down to the bottom of the chimney which then Smoke
Precipitated ash

emits out only gaseous mixture.


Figure: The Cottrell smoke precipitator

Formation of the Delta: as the mouth of the river is an example of coagulation. Rivers
carrying muddy water as suspension when come near sea, the high salinity (salt content) of the
sea cause suspension to coagulate forming delta. Artificial rain can be made by coagulating the
water containing clouds in the sky.
One can endlessly quote the application where the stability or instability of colloidal dispersions
is desired.
Rubber Plating:The negatively charged rubber particles from the latex are deposited on
wares and handles of different tools. Ruber gloves are formed by rubber plating on
suitable templates.
In nontechnology: nanosized materials are prepared for use as catalyst by using
reverse micelles.
In metallurgy:The metal sulphide ores are concentrated by froth-floatation process
which involves the treatment of the pulverized ore in emulsion of pine oil.
In road construction:Asphalt in water emulsion is used for building roads without the
necessity of melting the asphat.
PRACTICAL ORGANIC CHEMISTRY
Detection of Elements in Organic Compound: Besides carbon and hydrogen, an
organic compound may also contain elements like nitrogen, sulphur, halogens, oxygen and
sometimes metals and other non-metals.
Detection of carbon and Hydrogen: The compound is mixed with dry cupric oxide (CuO)
and strongly heated. C and H present in the compound gets oxidized to CO2 and H2O
respectively. CO2 turns lime water milky which dissolves in excess to form Ca(HCO3)2, whereas
water vapours turn anhydrous CuSO4 to blue.

C + 2CuO  → CO 2 + 2Cu
CO 2 + Ca(OH) 2 
→ CaCO3 ↓ + H 2 O
(White)

CaCO3 + H 2 O + CO2 
→ Ca(HCO3 )2
(So lub le)

H 2 + CuO 
→ H 2 O + Cu
CuSO4 + 7H 2 O 
→ CuSO4 + 7H 2 O
(White) (Blue vitriol)

Detection of Nitrogen, Halogens and Sulphur: These elements are detected by means of
Lassaigne test. The Lassaigne solution is prepared by fusing the organic compound with dry
sodium pieces (two or three such fusions are made). The fusions are added in distilled water
which is boiled and filtered off. The clear solution is known as Lassaignesolution (or sodium
extract). Under the fusion conditions the following reactions take place:
Na + C + N 
→ NaCN
Sodium cyanide

Na + Cl 
→ NaCl
Na + Br 
→ NaBr
Na + I 
→ NaI
2Na + S 
→ Na 2S

Unreacted sodium is converted into NaOH solution.


Detection of Nitrogen: To a little sodium extract (Lassaigne solution) we add freshly
prepared ferrous sulphate solution. The solution is made alkaline and heated. On addition of dil.
H2SO4, if a green or blue colour appears, nitrogen is present. If no appearance of colour, add a
little ferric ion solution. A blue or green colour of ferric ferrocyanide confirms nitrogen.

FeSO 4 + 2NaOH 
→ Fe(OH)2 ↓ + Na 2SO 4
(Green ppt.)

Fe(OH) 2 + 6NaCN 
→ Na 4 [Fe(CN)6 ] + 2NaOH
Sodium ferrocyanide

3Na 4 [Fe(CN) 6 ] + 2Fe 2 (SO 4 )3 


→ Fe 4 [Fe(CN) 6 ]3 + 6Na 2SO 4

or 2Na 4 [Fe(CN)6 ] + 4FeCl3 


→ Fe4 [Fe(CN)6 ]3 + 12NaCl
Ferric ferrocyanide
(Blue)
If the compound contains both nitrogen and sulphur then there will be sodium sulpho or
thiocyanide in the sodium extract. This on addition of FeCl3 will produce a red colour of ferric
sulphocyanide.

Na + S + C + N 
→ NaSCN
3NaSCN + FeCl3 
→ Fe(SCN)3 + 3NaCl
Ferric sulphocyanide
(Red colour)

Diazo compound (—N=N—) do not give test of nitrogen as above, but a mixture of potassium
carbonate and magnesium powder is used in place of sodium for the preparation of extract.
Soda lime test for nitrogen:Many organic compounds on heating with soda lime (CaO +
NaOH) give smell of ammonia.

CH3CONH 2 + NaOH 


CaO

→ CH 3COONa + NH 3 ↑
Acetamide

NH 2 CONH 2 + 2NaOH 


CaO

→ Na 2 CO3 + 2NH 3 ↑

Nitro and diazo compounds do not give ammonia on heating with soda lime.
Detection of Halogens

Lassaigne test:The extract is boiled with conc. AgNO3 to expel nitrogen and sulphur, because
NaCN and Na2S react with AgNO3 to give white and black precipitates, respectively.

NaCN + AgNO3 
→ AgCN ↓ + NaNO3
Na 2S + 2AgNO3 
→ Ag 2S ↓ + 2NaNO3

Boiling with conc. HNO3 evolves HCN and H2S as volatile gases.

NaCN + HNO3  → NaNO3 + HCN ↑
(Conc.)

Na 2S + 2HNO3  → 2NaNO3 + H 2S ↑
(Conc.)

Now add AgNO3 solution.


i) Formation of a curdy white precipitate which is soluble in NH4OH solution shows the
presence of chlorine.

NaCl + AgNO3 
→ AgCl ↓ + NaNO3
(White)

ii) Formation of a light yellow ppt, which is sparingly soluble in NH4OH shows the presence
of iodine.
NaI + AgNO3 
→ AgBr ↓ + NaNO3
(Light yellow )

iii) Formation of a lemon yellow ppt. insoluble in NH4OH shows the presence of iodine.
NaI+ AgNO3 
→ AgI ↓ + NaNO3
(Lemon yellow )
Layer test for bromine and iodine:The presence of nitrogen and sulphur does not interfere
in this test. In this test acidify a little sodium extract with dil. HNO3 or dil. H2SO4 and add CHCl3
or CCl4 and excess of chlorine water. If organic layer becomes yellow or brown, bromine is
present and if violet, iodine is present.
2NaBr + Cl2 
→ 2NaCl + Br2
CHCl3 + Br2 
→ Yellow or brown layer
2NaI + Cl2 
→ 2NaCl + I 2
CHCl3 + I 2 
→ Violet layer

Beilstein’s Test:Many organic compounds containing halogens impart a green colour to the
Bunsen flame when heated on a copper wire. The green colour is developed due to the formation
of volatile halides of copper. However, nitrogen compound urea also imparts green colour to the
flame.
Detection of Sulphur: The test of sulphur from sodium extract many be done in three ways:

i) Sodium nitroprusside test:Only a little reagent is added to sodium extract;


appearance of purple, pink or violet colour confirms the presence of sulphur.

Na 2 [Fe(NO)(CN)5 ]+ Na 2S 
→ Na 3 [Fe(O = N — S — Na)(CN)5 ]
Sodium nitroprusside Violet complex

ii) Lead acetate test:Add a few drops of lead acetate solution to sodium extract;
appearance of a black ppt. shows the presence of sulphur.

(CH 3COO) 2 Pb + Na 2S 
→ 2CH3COONa + PbS ↓
(Black )

iii) Silver coin test:On adding a silver coin to the sodium extract, the coin turns black.

iv) Oxidation test of sulphur:The unknown compound is heated with fusion mixture
(Na2CO3 + KNO3) in an ignition tube when sulphur is oxidized to sulphate.

Na 2 CO3 + S + 3[O] 
→ Na 2SO 4 ↓ + CO 2 ↑

The fused mass is extracted with hot water, filtered and a few drops of barium chloride
solution are added. The appearance of a white precipitate which is insoluble in conc. HCl
or HNO3, confirms the presence of sulphur.

Na 2SO4 + BaCl2 
→ BaSO4 ↓ + 2NaCl
(White)

BaSO 4 + HNO3 
→ Inso lub le
(Conc.)

Test of halogen present in the ionic form:Benzene diazonium halides and halide salts of
aliphatic and aromatic amines contain halide ions. If the aqueous solution of any compound gives
a white or yellow ppt. on addition of AgNO3, halide ions are present.
C6 H 5 N +2 Cl – + AgNO3 
→ C6 H 5 N +2 NO3– + AgCl ↓
Benzene diazonium
chloride
C6 H 5 N 2 Br + AgNO3 
→ C 6 H 5 N 2 NO3 + AgBr ↓

C6 H5 NH 3+ Cl – + AgNO3 
→ C6 H 5 NH3+ NO3– + AgCl ↓
Aniline hydrochloride

Detection of sulphur present in the side chains: If the addition of BaCl2 solution to
aqueous solution of organic compound gives white precipitate, it indicates the presence of
sulphate ion in it.

C6 H5 N 2 HSO4 + BaCl2 
→ BaSO 4 ↓ + HCl + C6 H 5 N 2 Cl
Diazonium Sulphate

(C6 H5 NH 3 ) +2 SO 2– → 2C6 H 5 NH 3+ Cl – + BaSO 4 ↓


4 + BaCl 4 
Anilinium sulphate

Detection of Oxygen: There is no satisfactory qualitative test for oxygen. If the compound on
heating in an atmosphere of hydrogen gives water drops, oxygen may be present.

Summary of SolubilitiyBehaviour
It is convenient to summarize the solubility characteristics of the common classes of organic
compounds into seven groups as specified in Table 1. The compounds are grouped according to:
(a) their solubility towards the reagents specified above, and (b) the elements, other than carbon
and hydrogen, that they contain.

1. Compounds soluble in both water and ether:This includes the lower


members of the various homologous series (4-5 atoms in a normal chain) that contain
oxygen and/or nitrogen in their structures: they are soluble in water because of their low
carbon content. If the compound is soluble in both water and in ether, it would also be
soluble in other solvents so that further solubility tests are generally unnecessary: the
aqueous solution should be tested with indicator paper. The test with sodium hydrogen
carbonate solution should also be performed.

2. Compounds soluble in water but insoluble in ether:These compounds with


the exception of salts, are usually also soluble in dilute alkali and acid. The behaviour of
salts to alkaline or acidic solvents may be informative. Thus, with a salt of a water-
soluble base, the characteristic odour of an amine is usually apparent when it is treated
with dilute alkali: likewise, the salt of a water soluble, weak acid is decomposed by dilute
hydrochloric acid or by concentrated sulphuric acid. The water-soluble salt of a water-
insoluble salt of a water-insoluble acid or base will give a precipitate of either the free
acid or the base when treated with dilute acid or dilute alkali. The salts of sulphonic acids
and of quaternary bases are unaffected by dilute sodium hydroxide or hydrochloric acid.

3. Compounds insoluble in water, but soluble in dilute sodium


hydroxide:It should be recalled that some of the compounds belonging to this group
are sufficiently strong acid to release carbon dioxide from sodium hydrogen carbonate
(e.g. carboxylic acids, sulphonic and sulphinic aids, and certain substituted phenols, see
below).
Table 1: Classification of organic compounds according to solubility behaviour

(1) (2) (3) (4) (5) (6) (7)

Not containing N Not containing N


Soluble in water Soluble in 5% Neutral
Soluble in both ether Soluble in 5% or S. Soluble only or S. Insoluble in
but insoluble in sodium hydroxide compounds
and water hydrochloric acid in concentrated concentrated
ether solution containing N or S
sulphuric acid sulphuric acid

The lower members of Polybasic acids and Acids. Phenols. Primary amines. Unsaturated Saturated aliphatic Nitro compounds
the homologous series hydroxyl acids. Imides. Some Secondary aliphatic hydrocarbons. hydrocarbons. (tertiary). Amides
of; Alcohols; Acids; Glycols, polyhydric primary and and aryl-alkyl Some polyalkylated Cycloalkanes. and derivatives of
Esters; Phenols ; alcohols, secondary nitro amines. Aliphatic aromatic Aromatic aldehydes and
Anhydrides; Amines; polyhydroxy compounds; and some aryl-alkyl hydrocarbons. hydrocarbons. ketones. Nitriles.
Nitriles; aldehydes and oximes. Thiols and tertiary amines. Alcohols. Halogen derivatives Negatively
Polyhydroxyphenols. ketones (sugars). thiophenols. Hydrazines. Aldehydes. of the above substituted amines.
Some amides, Sulphonic acids, Ketones. Esters. hydrocarbons, Nitroso, azo,
amino acids, di- and sulphinic acids, Anhydrides. Etehrs Diaryl ethers. hydrazo and other
polyamino aminosulphonic and acetals. intermediate
compounds, amino acids and Lactones. Acyl reduction products
alcohols. Sulphonic sulphonamides. halides. of nitro compounds.
acids. Salts. Some diketones and Sulphones,
β-keto esters. sulphonamides of
secondary amines,
sulphides, sulpahtes
and other sulphur
compounds.
4. Compounds insoluble in water, but soluble in dilute hydrochloric
acid:It should be remembered that the hydrochlorides of some bases are sparingly
soluble in cold water and one should therefore not be misled by an apparent insolubility
of a compound (containing nitrogen) in dilute hydrochloric acid. The suspension in dilute
hydrochloric acid should always be filtered and the filtrate made alkaline. A precipitate
will indicate that the compound is indeed a base and should be included in this group.

5. Water-insoluble hydrocarbons and oxygen compounds that do not


contain N or S and are soluble in cold concentrated sulphuric acid:
Any changes – colour, excessive charring, evolution of gases or heat, polymerization and
precipitation of an insoluble compound – attending the dissolution of the substance
should be carefully noted.
Alcohols, esters (but not ethyl benzoate, diethyl malonate or diethyl oxalate), aldehydes,
methyl ketones and cyclic ketones containing less than nine carbon atoms as well as
ethers containing less than seven carbon atoms are also soluble in 85 per cent phosphoric
acid.

6. Compounds, not containing N or S, insoluble in concentrated


sulphuric acid: This test provides for a differentiation inter alia between alkanes and
cylco-alkanes and also simple aromatic hydrocarbons which are insoluble, and
unsaturated hydrocarbons which are soluble in the reagent.

7. Compounds that contain N or S which are not in groups 1-4: many of


the compounds in this group are soluble in concentrated sulphuric acid. It will be
observed that halogen compounds are not listed separately, but appear in each of the
seven categories in accordance with their solubility behaviour.
Mind Map
Mixture

Distilled on water-bath

Non-volatile Volatile (I)


treated with ether

Insoluble in ether (II) Soluble in ether treated


with dilute H2SO4

Neutral or acidic treted Basic (III)


with dilute NaOH

Neutral treated with Acidic treated with


NaHSO4 CO2

Aldehydes and Neutral (V) Phenols (VI) Acids and


ketones (IV) Nitrophenols (VII)
Detection of Functional Group
Test for Alcoholic Hydroxyl Group

1. Sodium Test:to 1 mL of aqueous (or alcoholic) solution of the compound add a few
drops of ferric chloride solution. A violet, red, blue or green coloration indicates the
presence of phenolic group.

2. Ceric Ammonium Nitrate:to 1 mL of aqueous hot solution of the compound, add a


few drops of ceric ammonium nitrate. A green or brown precipitates indicates the
presence of ––OH group.

3. Molybdate Test:Heat a little of the organic compound with 2 pellets of KOH. Cool and
add 1 mL of ether and 2-3 drops of CS2. A yellow precipitate will be obtained. Filter the
solution and to the filtrate add 1 mL of ammonium molybdate solution and excess of
dilute HCl. A red or blue coloration indicates the presence of ––OH group.

Test Phenolic Hydroxyl Group

1. Ferric Chloride Test:To 1 mL of aqueous (or alcoholic) solution of the compound add
a few drops of ferric chloride solution. A violet, red, blue or green coloration indicates the
presence of phenolic group.

2. Ceric Ammonium Nitrate Test:To 1 mL of aqueous hot solution of the compound,


add a few drops of ceric ammonium nitrate. A green or brown precipitates indicates the
presence of phenolic group.

3. Phthalein Test:Heat about 0.1g of the compound with 0.1g phthalic anhydride and few
drops of concentrated H2SO4. Cool, pour the solution in dilute KOH solution. A red, blue
or fluorescent green coloration indicates the presence of phenolic group.

Test of Aldehyde and Ketonic Groups

1. 2,4-Dinitrophenylhydrazine Test:Shake 2-3 drops of alcoholic solution of organic


compound with 1 mL of 2,4-dinitrophenylhydrazine reagent. A yellow or orange
precipitate indicates the presence of carbonyl group.

2. Sodium Bisulphite Test:Shake a little organic compound (say, 0.2g or 4-5 drops)
with 1 mL of a saturated solution of sodium bisulphate. A crystalline precipitates
indicates the presence of carbonyl group.

Test for Aldehydic Group

1. Tollens Reagent Test:To a little of the organic compound in a test tube add a 1 mL of
Tollens reagent (ammonical solution of silver nitrate). Warm and allow to stand. The
appearance of silver mirror at the sides of test tube indicates the presence of aldehydic
group.

2. Fehling’s Solution Test:Warm a little of organic compound with 1 mL of Fehling’s


solution. Appearance of red precipitate indicates the presence of aldehydic group.
3. Schiff’s Reagent Test:To a little of organic compound add a few drops of Schiff’s
reagent. Appearance of pink or violet coloration indicates the presence of aldehydic
group.

Test for Ketonic Group

If a carbonyl group is present, the absence of aldehydic group tests indicates the presence of
ketonic group.

Tests for Carboxylic Group

1. Bicarbonate Test:To a little of organic compound add2-3 mL of 5% NaHCO3


solution. Brisk appearance of CO2 indicates the presence of ––COOH group.

2. Ester Test:Warm a little organic compound add 2-3 drops of concentrated H2SO4 and 2
mL of alcohol. A fragrant smell indicates the presence of ––COOH group.

Test for Primary Amine Group

1. Isocyanide Test:Warm a little of organic compound with 2-3 drops of chloroform and
1 mL of alcoholic KOH. A very unpleasant odour of isocyanide indicates the presence of
––NH2 group.

After performing the test, add 2-3 mL of concentrated HCl to destroy isocyanide.

2. Dye Test for aromatic primary amines:Cool in ice 0.2g (or 4-5 drops) of the
organic compound with 1 mL concentrated HCl. Add 2 mL of iced-cold dilute NaNO2
solution. Shake and add-23 mL of alkaline β-napthol solution. A red or orange dye
indicates the presence of aromatic ––NH2 group.

Test for Secondary Amino Group

Liebermann’s Nitroso Reactions: Cool in ice 0.2g (or 4-5 drops) of the organic compound
with 10 mL of dilute HCl. Add 2 mL of iced-cold dilute NaNO2 solution. A white or yellow oily
emulsion is obtained. Take 2-3 drops of this emulsion and add 3-4 drops of phenol and 3-4 drops
of concentrated H2SO4. Warm the solution. A blue coloration which changes to red on adding
1-2 mL of water. The colour changes to blue on adding a few drops of NaOH solution.

Test for Nitro Group

1. Dye Test: To a little or organic compound add 1 mL of concentrated HCl and 3-4 small
pieces of granulated tin. Heat the mixture for 5 minutes in a boiling water bath. Filter the
solution and cool the filtrate in ice-cold water. Add 1-2 mL of NaNO2 solution followed
by 2 mL Of alkaline β-naphthol solution. The appearance of red or orange coloured dye
indicates the presence of ––NO2 group.

2. Mulliken and Baker’s Test:Dissolve a little of organic compound in about 1 mL


alcohol. Add about 1 mL of NH4Cl solution and a pinch of Zn dust. Warm the mixture
and filter directly in 2 mL of Tollens reagent. Appearance of grey or black precipitates
indicates the presence of ––NO2 group.
Methods of Separation of Mono-functional Organic Compounds from
Binary Mixtures

Separation Based on Differences in the Chemical Properties of the Compounds


1. Toluene and Aniline:A mixture of toluene may be separated by extraction with dilute
HCl; the aniline passes into the aqueous layer in the form of the salt aniline hydrochloride
and may be recovered by neutralization.
2. Phenol and Toluene:They can be separated by treatment with dil. NaOH
3. Dibutyl Ether and Chlorobenzene:Concentrated H2SO4 dissolves only the dibutyl
ether and it may be recovered from solution by dilution with water.
The above examples are simple applications of the fact that the various components fall
into different solubility groups.
O
||
4. Phenols and any compound containing - C - OH group (e.g. o-cresol from
benzoic acid):Phenols may be separated from acids by a dilute solution of NaHCO3; the
weakly acidic phenols (and also enols) are not converted into salts by this reagent and
removed by ether extraction; the acids pass into solution as the sodium salts and may be
recovered after acidification.
5. Separation of mixture of amines:Mixture of primary, secondary and tertiary
amines can usually be separated by Hinsberg method. The mixture of amines is treated
with NaOH solution and toluene-p-sulphonyl chloride is warmed in a water bath and
finally acidified with dil. HCl, the sulphonamides of primary and secondary amines are
precipitated. The solid is filtered and washed with a little cold water, the tertiary amine
will be present in the filtrate. The solid is then treated with alkali again. The
sulphonamide from the primary amine goes into solution and the precipitate of the
sulphonamide of the secondary amine is filtered off. The filtrate is acidified with dil. HCl
to precipitate the derivative of the primary amine.
6. Aldehydes and Ketones from other liquid hydrocarbons and other neutral
liquid compound (e.g. ethanal and ethanol):The mixture is shaken with a
solution of NaHCO3, the carbonyl compound forms a solid bisulphite compound which is
filtered off and decomposed with dil. acid in order to recover the aldehyde or ketone.

Separation based on Differences in the volatilities of the component in aqueous


solution
1. Diethylamine and butan-1-ol: They can be separated by adding sufficient dilute
H2SO4 to neutralise the base; steam distillation will remove the alcohol. The amine can be
recovered by adding NaOH to the residue and repeating the distillation.
2. Diethyl Ketone and Acetic Acid:The mixture is treated with sufficient dilute NaOH
solution to transform the acid into sodium acetate and distilling the aqueous mixture. The
ketone will pass over in the steam and the non-volatile stable salt will remain in the flask.
Acidification with dilute H2SO4 liberates acetic acid which can be isolated by steam
distillation or by extraction.
Determination of Empirical Formula and Molecular Formula
Determination of Empirical Formula:The empirical formula of a molecule is
determined using the % of elements present in it./ Following method is adopted.

Element % Relative No. of atoms = % /at. wt. Simplest Ratio Empirical formula

Relative No. of Atoms:Divide the percentage of each element present in compound by its
atomic weight. This gives the relative number of atoms of element in molecule.

Simplest Ratio:Find out lowest value of relative number of atoms and divide each value of
relative number of atoms by this value to get simplest ratio of elements.

If the simplest ratio obtained are not complete integers, multiply them by a common factor to get
integer values of simplest ratio.

Empirical Formula:Write all constituent atoms with their respective number of atoms
derived in simplest ratio. This gives empirical formula of compound.

Molecular Formula:Molecular formula = n × empirical formula where ‘n’ is the whole


number of obtained by

molecular weight of compound


n=
empirical formula weight of compound

molecular weight of organic compound can be determined by any method for which data is
given.
Solved Problems
Objective
Problem 1: At 400K energy of activation of a reaction is decreased by 0.8 kcal in presence of
catalyst. Hence rate will be
(a) Increased by 2.71 times (b) Increased by 1.18 times
(c) Decreased by 2.72 times (d) Increased by 6.26 times

Solution: (a) K1 (no catalyst) = Ae − Ea / RT


K2 (catalyst) = Ae − E a −0.8 / RT
K2
= e0.8 / RT = e0.8 / 0.002×400 2.72
K1

Problem 2: A white precipitate of Sn(OH)4 is peptized with dil. HCl. The sol particle will
carry:
(a) Positive charge
(b) Negative charge
(c) Sometimes positive and sometimes negative charge
(d) No charge

Solution: (a) Zn(OH)4 + 4HCl → SnCl4 + 4H2O


[Zn(OH)4]Sn4+ M 4Cl– (Positive charge)

Problem 3: Which of the following reactions leads to the formation of a substance in colloidal
state?
(a) 2HNO 3 + 3H 2S 
→ 3S + 3H 2 O + 2NO
(b) SnCl2 + HgCl2 
→ SnCl4 + Hg
(c) 2Mg + CO 2 
→ 2MgO + C
(d) Cu + CuCl 2 →
HCl
Cu 2 Cl2

Solution: (a) Due to oxidation of H2S by HNO3 colloidal particles of sulphur (S8) are
formed. In other cases, particles of colloidal size are not formed.

x
Problem 4: Graph between log and log P is a straight line inclined at an angle θ - 45°.
m
When pressure of 0.5 atm and log k = 0.699, the amount of solute adsorbed per g
of adsorbent will be
(a) 1 g/g adsorbent (b) 1.5 g/g adsorbent
(c) 2.5 g/g adsorbent (d) 0.25 g/g adsorbent

x
Solution: (c) = k ⋅ P1/ n since log k = 0.699, hence k = 5
m
hence k = 5
1 x
slope = = tan 45° = 1, thus, = 5 × 0.5 = 2.5 g/g adsorbent
5 m
Problem 5: For adsorption, the thermodynamic requirement is that:
(a) ∆H must be negative (b) ∆S must be negative
(c) ∆G must be negative (d) ∆H, ∆S and ∆G must be negative

Solution: (d) On the basis of equation, ∆G = ∆H – T∆S can be negative if ∆H has


sufficiently high negative value as –T∆S is positive. Thus, in adsorption which
is a spontaneous process.
∆S is negative
∆H is sufficiently negative and as a result ∆G is also negative

Problem 6:: 1 mol of [AgI]I– can be coagulated by


1
(a) 1 mol of Pb(NO3)2 (b) mol of Pb(NO3 ) 2
2
2
(c) mol of Pb(NO3 ) 2 (d) None of these
3

Solution: (b) Pb(NO 3 ) 2 + 2[Ag[I − 


→ PbI 2 ↓ + AgI ↓
2[Ag]I − ≡ Pb(NO 3 ) 2
1
∴[Ag]I− ≡ Pb(NO3 )2
2

Problem 7: The process of converting precipitate into collidal solution on adding an


electrolyte is called:
(a) Peptisation (b) Dialysis
(c) Electro-osmosis (d) Eelectrophoresis

Solution: (a) The conversion of a fresh ppt. into colloidal state by addition of small amount
of a suitable electrolyte is called peptization e.g.
Fe3+
Fe 3+
Fe3+
Cl−
3+ − 3+
Fe(OH)3 + Fe + 3Cl 
→ Fe Fe(OH)3 Fe 3+
Cl−
Cl−
Fe3+
Fe 3+
Fe3+
i.e., ions are adsorbed on particles to give colloidal size.

Problem 8: The Brownian movement of colloidal particles is because of


(a) Convection currents in the fluid
(b) Unequal bombardments by the molecules of the dispersion medium on
colloidal particles.
(c) Setting of dispersed phase under gravity
(d) Thermal gradient in the medium

Solution: (a) Unequal bombardments of solvent particles on colloidal particles cause


zig-zag motion.
Problem 9: The stabilization of the dispersed phase in a lyophobic sol is due to
(a) The viscosity of the medium
(b) The surface tension of the medium
(c) Liking for the medium
(d) The formation of an electrical layer between the two phases

Solution: (d) Due to preferential adsorption of similar ions, an electrical double layer
formed on the surface of colloidal particles e.g., arseniussulphide solution.
S′′
S′′ S′′
H+
S′′ Ag 2S3 S′′ H +
H+
S′′ S′′
S′′

Problem 10: When FeCl3 solution is added to NaOH a negatively charged sol is obtained. It is
due to the
(a) Presence of basic group
(b) Preferential adsorption of OH– ions
(c) Self dissociation
(d) Electron capture by sol particles
Solution: (b) FeCl2 + 3NaOH  → Fe(OH)3 + 3NaCl
NaOH  → Na + + OH −
OH– ions having similarity with Fe(OH)3will be adsorbed on the surface of
Fe(OH)3 to form an electrical double layer.
OH − Na +
Fe(OH)3 OH − Na +
OH − Na +

Subjective

Problem 1: A mixture of p-amino benzoic acid and p-hydroxy benzoic acid is taken in
diethyl ether. How will you separate them (in not more than three steps)?
Write the reagents and all the required conditions to separate them. Given
confirmatory test for the presence of each functional group.

Solution: p-amino benzoic acid,


p-hydroxy benzoic acid (in
ether)
Step 1. Extract
with aq. dil. HCl

Ether layer Aqueous layer

p-HO-C6H4CO2H p-CO2H-C6H4NH3+Cl-

Separate the two layers using separating funnel.


Step 2: Basification of aqueous layer with NaOH.
Test for –NH2 group: When compound containing –NH2 group is heated with
CHCl3 and KOH, a very pungent smell of isocyanide is obtained.
RNH2 + CHCl3 + 3KOH → RNC + 3KClk + 3H2O
Test for –COOH group: The compound containing –COOH group gives
brisk effervescence on treatment with NaHCO3 solution due to evolution of
CO2.
Test for –OH group: A blue or green coloration obtained when the
compound is treated with neutral FeCl3 solution.

Problem 2: Artificial rain can be caused by spraying charged dust particles over
colloids.
Solution: Clouds represent the colloidal solutions of water drops in air (liquid in gas
type). These drops are expected to carry some charge (positive or negative). In
order to neutralize the charge on these, charged dust particles carrying
opposite charge are sprayed over a certain layer of cloud. These will neutralize
the charge on water droplets resulting in their coagulation. The bigger water
drops can no longer be retained by the atmosphere and will result in the
artificial rain.

Problem 3: Ferric hydroxide sol is more readily coagulated by Na3PO4 in comparison


to KCl.

Solution: Ferric hydroxide sol is positively charged and to cause its coagulation, ions
carrying negative charge are needed. Since PO 3–4 ions have higher negative

charge than Cl ions therefore, Na3PO4 coagulates the ferric hydroxide sol
most efficiently.
Problem 4: Delta is generally formed when river meets the ocean.
Solution: River water is generally muddy and carries along with colloidal dust particles
which are charged in nature. Sea water contains a large number of electrolytes.
When the river comes in contact with the sea water, the colloidal particles get
their charge neutralized by the oppositely charged ions present in sea water
and are coagulated. This ultimately results a hard solid mass known as delta.
Problem 5: The layer of fat in the pans used for preparing soaps can be removed by
adding boiling washing soda solution. How will you account for it?
Solution: Washing soda (Na2CO3) gets hydrolysed to form NaOH as follows:
Na 2 CO 3 + H 2 O 
(boil)
→ 2NaOH + H 2 O + CO 2
NaOH reacts with fat which is a triglyceride to form soap and glycerol by
saponification reaction. The soap thus formed helps in cleaning the pan.
Problem 6: SnO2 forms a positively charged colloidal sol in the acidic medium and
negatively charged sol in the basic medium. Explain.

Solution: SnO2 is of amphoteric in nature which means that it can react with both acis
and bases. With an acid such as HCl, if forms SnCl4. The Sn4+ ions are
adsorbed on the surface of SnO2 to give a positively charged colloidal sol.
SnO 2 + 4HCl 
→ SnCl4 + 2H 2 O
SnO 2 + Sn 4 + 
→ [SnO 2 ]Sn 4+
Positively ch arg ed

Similarly with base like NaOH, if forms sodium stannate (Na2SnO3). The
stannate ions get adsorbed on the surface of SnO2 to give negatively charged
colloidal sol.
SnO 2 + 2NaOH 
→ Na 2SnO 3
SnO2 + SnO32– 
→ [SnO 2 ]SnO32–
negatively ch arg ed

Problem 7: Gelatin is generally added to ice creams.

Solution: Ice cream is an emulsion of milk or cream in water (oil in water). Gelatin is
added in the preparation of ice cream to act as emulsifier i.e., it helps in the
formation of a stable emulsion.

Problem 8: The addition of ferric hydroxide solution to arseneoussulphide solution results


in the precipitation of both.

Solution: This is possible only in case equimolar solutions (solutions with same number
of moles) are mixed. Since the ferric hydroxide sol carries positive charge and
arsenic oxide sol is negatively charged on mixing they will get their charge
neutralized and will be coagulated.

Problem 9: Lyophilic solar are called reversible colloids.

Solution: Lyophilic sols are generally known as reversible colloids. In fact, if th


dispersed phase is removed completely the colloidsl solution can be formed
again by mixing the dispersed phase (residue) left in a fresh sample of
dispersion medium. for example, if a colloidal sol of starch in water is dried
completely, it can be reformed by mixing the residue with fresh water.

Problem 10: In fine weather, we normally see red colour of setting sun. Can you co-relate
this with the colloids?

Solution: We all know that dust particles are present in the atmosphere. When light
emitted by the setting sun passes through the blanket of dust, the blue part of
the light is scattered away from our eyes and at the same time the red colour is
seen. Thus, the setting sun has red colour.

Problem 11: Which out of the following solutions having the same concentration will be
most effective in causing the coagulation of the arsenic sulphide sol which is
yellowing colour: KCl, MgCl2, AlCl3 or Na3PO4 ?

Solution: Arsenic sulphide (As2S3) sol has a negative charge on it. To cause it
coagulation or precipitation the active ions must be positively charged.
According to hardy-Schulze rule, greater the magnitude of the positive charge
on the ion more will be its coagulating power. Thus, AlCl3 containing Al3+
ions will be the most effective in causing the coagulation of the sol.
Problem 12: The coagulation values of the electrolytes AlCl3and NaCl for As2S3 sol are
0.093 and 52 respectively. How many times AlCl3 has greater coagulating
power than NaCl?

Solution: It may be noted that the coagulating powers of the electrolytes are inversely
proportional to their coagulating values. Thus,
Coagulatign power of AlCl3 52
= = 559
Coagulating power of NaCl 0.093
Thus, coagulating power of AlCl3 is 559 times more than that of NaCl.

Problem 13: Draw a flow diagram for the separation and recovery in almost quantitative
yield of a mixture of the water – insoluble compounds PhCHO, PhNMe2,
PhCl, p-MeC6H4OH and PhCOOH.

Solution: Ether solution PhCHO,


PhNMe2, PhCl, p-MeC6H4OH
and PhCOOH.
Extract with dilute
aqueous HCl.

Lower aqueous HCl layer Upper ether layer

PhMe2H+Cl- add. Extract with aqueous


aq. NaOH, ether NaOH
Lower NaOH layer

PhNMe2 in ether p-MeC6H4O-Na+, PhCOO-Na+ Add aq. NaHSO3


add dry ice (CO2), ether

Lower aq. NaHCO3 layer Upper ether layer Lower aq. layer Upper ether layer

PhCOO-Na+ add di. HCl, ether

PhCOOH in ether p-MeC6H4OH in ether PhCHO in ether PhCl in ether


KVPY
1. An organic compound fused with metallic sodium dissolves in water and the solution is divided into
two parts. One part is treated with FeSO4, boiled and filtered. In the filtrate addition of FeCl3 does not
produce any precipitate. To the other part, addition of sodium nitro prusside produces violet color. The
organic compound contains. (2007)
(A) only nitrogen (B) both bromine and nitrogen
(C) only sulphur (D) both sulphur and nitrogen
Ans:C

2. Copper in an alloy is estimated by dissolving in conc. nitric acid. In this process copper is converted to
cupric nitrate with the evolution of nitric oxide (NO). The misture when treated with potassium iodide
forms cupric iodide. Which is unstable and decomposes to cuprous iodide and iodine. The amount of
copper in the alloy is estimated by litrating the libereated iodine with sodium thiosulphate. The
reactions are (2010)
a Cu + b HNO3 → c Cu(NO3 )2 + d NO + eH2 O
f Cul2 → gCu2I2 + hI2
i Na2 S2O3 + ˆjI2 → k Na2 S4 O6 + INal
(Fill up the blanks)
(A)The coefficients are : a = 3 .b = 8 ,c = 3 ,d = 2 and e = 4
(B)The coefficients are : f = 2 ,g =1 and h = 1
(C)The coefficients are : i = 2 , j =1 ,k =1 and I = 2
(D)If 2.54 g of I2 is evolved from a 2.0 g sample of the alloy, what is the percentage of copper in the
alloy?
(atomic, weight of iodine and copper are 127 and 63.5, respectively) = 63.5%
Ans:

3. A water droplet spreads on a clean gold surface. The droplet, however, does not spread onto a gold
surface which is pre-treated with a dilute solution of hexanethiol CH3 − ( CH2 )5 − SH and washed
with excess water. This is because (2007)
(A) chemisorption of hexanethiol on gold surface renders it hydrophilic
(B) chemisorption of hexanethiol onto gold surface renders it hydrophobic
(C) physisorption of hexanethiol onto gold surface renders it hydrophobic
(D) physisorption of hexanethiol onto gold surface renders it hydrophilic.
Ans:B

Assignments
SECTION – I Single Choice Questions
1. The size of particles in suspension, true solution and colloidal solution varies in the order
(a) Suspension > Colloidal > True Solution (b) True Solution > Suspension > Colloidal
(c) Suspension > Colloidal = True Solution (d) None of the above
2. Out of the following which reaction gives a colloidal solution?
(a) Cu + HgCl2→ CuCl2 + Hg (b) 2HNO3 + 3H2S → 3S + 4H2O + 2NO
(c) 2Mg + CO2→ 2MgO + C (d) Cu + CuCl 2  → Cu 2 Cl 2
(in presence of excess of HCl)

3. Colloidal solution of silver is prepared by


(a) Colloidal mill (b) Double decomposition method
(c) Bredig’s method (d) Peptization
4. Which is a natural colloid?
(a) Sodium chloride (b) Urea
(c) Starch (d) Barium nitrate
5. Which of the following is a homogeneous system?
(a) Muddy water (b) Brea
(c) Concrete (d) A solution of sugar in water
6. Emulsifiers are generally
(a) Soap (b) Synthetic detergents
(c) Lyophilic solutions (d) all of the above
7. The colloidal system consisting of a liquid as dispersed phase in a solid as dispersion
medium is termed as
(a) Aerosol (b) Gel
(c) Emuslion (d) Foam

8. Gels on standing exude small amounts of liquid. This phenomena is known as


(a) efflorescence (b) synersis
(c) thixotropy (d) adsorption
9. Substances whose solutions can readily diffuse through parchment membrane are
(a) Collods (b) Crystalloids
(c) Electrolytes (d) Non electrolytes
10. When dispersed phase is liquid and dispersion medium is gas then the colloidal system is
called
(a) Smoke (b) Clouds
(c) Jellies (d) Emulsions
11. The extra stability of lyophilic colloids is due to
(a) Charge on their particles
(b) A layer of medium of dispersion on their particles
(c) The smaller size of their particles
(d) The large size of their particles

12. Lyophilic solutions are more stable than lyophbic solutions because
(a) The colloidal particles have positive charge
(b) The colloidal particles have negative charge
(c) The colloidal particles are solvated
(d) The are strong electrostatic repulsions between the negatively charged colloidal
particles.

13. Which of the following is not a property of hydrophilic solutions?


(a) High concentration of dispersed phase can be easily attained
(b) Coagulation is reversible
(c) Viscosity and surface tension are nearly same as that of water
(d) The charge of the particle depends on the pH value of medium; it may be positive,
negative, or even zero

14. Flocculation value is expressed in terms of


(a) millimole L–1 (b) mole L–1
(c) gram L–1 (d) mole ml–1

15. When a colloidal solution is observed under an ultramicroscope, we can see


(a) Light scattered by colloidal particles (b) Size of the particle
(c) Shape of the particle (d) Relative size

16. The colour of the collidal particles of gold by different methods differ because of
(a) variablevalency of gold (b) different concentration of gold particles
(c) different type of impurities (d) different diameters of colloidal particles

17. On adding few drops of dil. HCl to freshly precipitated ferric hydroxide, a red coloured
colloidal solution is obtained. This phenomenon is known as
(a) Peptization (b) Dialysis
(c) Protective action (d) Dissolution

18. The ability of an ion to bring about coagulation of a given colloid depends upon
(a) Its size (b) The magnitude of its charge only
(c) The sign of its charge alone (d) Both magnitude and sign of its charge

19. A negatively charged suspension of clay in water will need for precipitation the minimum
amount of
(a) Aluminium chloride (b) Potassium sulphate
(c) Sodium hydroxide (d) Hydrochloric acid
20. The coagulation power of an electrolyte for As2S3 decreases in the order
(a) Na+, Al3+, Ba2+ (b) Cl–, SO 24 − , PO 34−
(c) Al3+, Ba2+, Na+ (d) PO 34− , SO 24 − Cl–

21. Which of the following pairs of ions would be expected to form precipitate when dilute
solutions are mixed?
(a) Na+, SO 32 − (b) NH +4 , CO 32 −
(c) Na+, S2– (d) Fe3+, PO 34−

22. Silver iodide is used for producing artificial rain because AgI
(a) is easy to spray at high attitudes (b) is easy to synthesize
(c) has crystal structure similar to ice (d) is insoluble in water

23. Which of the following electrolytes is least effective in causing flocculation of ferric
hydroxide solution?
(a) K3[Fe(CN)6] (b) K2CrO4
(c) KBr (d) K2SO4

24. Separation of colloidal particles from those of molecular dimension is known as


(a) Electrolysis (b) Electrophoresis
(c) Electrodialysis (d) None of the above

25. The blue colour of water in the sea is due to


(a) Refraction of the blue light by the impurities in sea water
(b) Reflection of blue sky by sea water
(c) Scattering of blue light by water molecules
(d) Absorption of other colours except the blue colour by water molecules

26. Distillation is used to separate liquids which differ in the b.p. by


(a) 5°C (b) 10°C
(c) 30–80°C (d) 100°C.
27. A mixture contains four solid organic compounds A, B, C and D. On heating only C
changes from solid to vapour state; C and be separated from the rest in the mixture by
(a) distillation (b) sublimation
(c) fractional distillation (d) crystallization
28. Which of the following techniques is most suitable for the identification of
cyclohexanone from a mixture containing benzoic acid, isoamyl alcohol, cyclohexane and
cyclohexanone?
(a) Crystallisation (b) IR spectroscopy
(c) Sublimation (d) Evaporation.

29. In sodium fusion test of organic compounds, the nitrogen of an organic compound is
converted to
(a) sodamide (b) sodium cyanide
(c) sodium nitrite (d) sodium nitrate.

30. A compound which does not give positive test for nitrogen is
(a) Urea (b) Azobenzene
(c) Glycine (d) Phenylhydrazine.

31. In Lassaigne’s test for nitrogen the blue colour is due to the formation of
(a) Ferric ferrocyanide (b) Potassium ferrocyanide
(c) Sodium ferrocyanide (d) Sodium cyanide.

32. Sodium extract of an organic compound gives blood red colour with FeCl3. It contains
(a) N (b) S
(c) N and S (d) S and Cl.

33. If N and S both are present in an organic compound, during Lassaigne’s test both changes
to
(a) Na2S and NaCN (b) NaSCN
(c) Na2SO3 and NaCN (d) Na2S and NaCNO.

34. The compound that does not give a blue colour in Lassaigne’s test is
(a) aniline (b) glycine
(c) hydrazine (d) urea
35. Liebig method is used for the estimation of
(a) nitrogen (b) sulphur
(c) C and H (d) halogens.

36. The Beilstein test for organic compounds is used to detect


(a) carbon (b) halogens
(c) nitrogen (d) sulphur.

37. In organic chemistry which element is always estimated by difference


(a) C (b) H
(c) O (d) N.

38. In Duma’s method, nitrogen present in organic compound is estimated as


(a) N2 (b) NO
(c) NH3 (d) H2.

39. In Kjeldahl’s method, nitrogen present is estimated as


(a) N2 (b) NH3
(c) NO2 (d) None of these.

40. The yield of organic reactions is generally poor because they are
(a) Accompanied by side reactions (b) Between covalent bonds
(c) Stoichiometric in nature (d) Very fast.
41. An organic compound contains about 52% carbon. It could be
(a) only ethanol (b) only dimethyl ether
(c) both (d) none of these.
42. The simplest formula of a compound containing 50% of element X (at. Wt. 10) and 50%
of element Y (at. wt. 20) is
(a) XY (b) XY2
(c) X2Y (d) X2Y2.
43. A dibasic acid containing C, H and O was found to contain C = 26.7% and H = 2.2%. The
vapour density of diethyl ester was found to be 73. What is molecular formula of acid?
(a) CH2O2 (b) C2H2O4
(c) C3H3O4 (d) C4H4O4.

44. Soda lime test is used to detect the following element of organic compound
(a) C (b) H
(c) N (d) S.

45. Aniline is purified by


(a) Steam distillation (b) Simple distillation
(c) Vacuum distillation (d) Extraction with a solvent.

46. A good solvent for sulphur is


(a) CCl4 (b) CS2
(c) H2O (d) toluene.
47. Anhydrous CaCl2 I used as drying agent because it
(a) absorbs water (b) adsorbs water
(c) absorbs as well as adsorbs water (d) none of these.

48. The most suitable method of separation of a 1 : 1 mixture of o- and p-nitrophenols is


(a) Sublimation (b) Chromatography
(c) Crystallisation (d) Steam distillation.

49. Which one of the following is not used for the purification of solid impurities?
(a) Distillation
(b) Sublimation
(c) Crystllisation
(d) All of these are used for the purification of solid impurities.

50. In a solution, solvent can be separated from solute by one of the following process
(a) Decantation (b) Filtration
(c) Distillation (d) Sedimentation

SECTION – II May be more than one choice

1. Adsorption is the phenomenon in which a substance:


(a) Accumulates on the surface of the other substance
(b) Can form chemical bond with molecules on surface of other substance
(c) Never form a chemical bond with other substance
(d) Remains close to the other substance

2. Physical adsorption is appreciable at:


(a) higher pressure (b) 100°C
(c) lower temperature (d) higher temperature

3. A poisonous gas is adsorbed at activated charcoal. Which among the following is true:
(a) Activated charcoal is absorber (b) Activated charcoal is adsorbate
(c) Activated charcoal has a porus surface (d) Activated charcoal is adsorbent

4. Which of the following is not a characteristic of chemisorption?


(a) Adsorption is non specific
(b) Adsorption is reversible
(c) Adsorption is independent of surface area
(d) Adsorption is specific

5. Which of the following is the application of adsorption?


(a) De-ionisation of water (b) Gas masks
(c) Hygroscopic nature of CaCl2 (d) Heterogeneous catalysis

6. Which of the following statements are correct?


(a) Physical adsorption is reversible in nature
(b) Physical adsorption involves vanderwaals forces
(c) Rate of physical adsorption increases with increase of pressure on the adsorbate
(d) High activation energy is involved

7. Which are not true about a catalyst?


(a) increase the equilibrium concentration of the product
(b) changes the equilibrium constant of the reaction
(c) shortens the time to reach equilibrium
(d) supplies energy to the reaction

8. Which of the following are accompanied by adsorption:


(a) decrease in entropy of the system
(b) decrease in enthalpy of the system
(c) T∆S for the process is negative
(d) None of these

9. Which plot among the following don’t represent adsorption isobar for chemi-sorption?

(a) (b)
x/m

x/m
T T

(c) (d)
x/m

x/m

T T

10. Which among the following are characteristic of chemi-sorption?


(a) Multilayer adsorption (b) exothermic in nature
(c) strong adsorption by free valencies (d) irreversible

11. Which of the following are correct?


x
(a) = constant (at high pressure)
m
x
(b) = constant X p1/n (at intermediate pressure)
m
x
(c) = constant X pn (at low pressure)
m
x
(d) = constant X p1/n (at high pressure)
m
12. The principle(s) not involved in chromatographic operation are:
(a) Partition (b) Absorption
(c) Adsorption (d) None of these
13. In adsorption, the forces associated can be:
(a) Ionic (b) Covalent
(c) Vanderwaals (d) H-bonding
14. Adsorption of gases on soid surface is generally exothermic because
(a) Free energy decreases (b) entropy decreases
(c) entropy increases (d) free energy increases
15. Which among the following property a gas should have to get adsorbed readily.
(a) higher critical temperature
(b) higher force of attraction among gaseous atoms
(c) higher atomic volume
(d) higher critical pressure
16. Which among the following are correct?
(a) All enzymes are protines (b) All proteins are not enzymes
(c) Enzymes are biocatalysts (d) All proteins are not enzymes

17. For adsorption which following conditions are favourable:


(a) ∆G < 0 (b) ∆S < 0
(c) ∆H > 0 (d) low temperature
18. Coagulation of colloids can be achieved by
(a) centrifugation (b) adding electrolyte
(c) change in pH (d) adding water

19. Which of the following are correct statements:


(a) Hardy schulz rule is related to coagulation
(b) Brownian movement and Tyndall effect are shown by colloids
(c) when liquid is dispersed in liquid, it is called gel
(d) gold number is a measure of protective power of lyophillic colloid
20. Which of the following are colloid
(a) smoke (b) Ruby glass
(c) pumic stone (d) chlorophyll
21. Colloidal solution can be purified by:
(a) Dialysis (b) Electrodialysis
(c) Electrophoresis (d) Ultrafiltration
22. Which of the following is/are +vely charged colloid.
(a) Al(OH)3 (b) SnO2
(c) Haemoglobin (d) Gold
23. Which of the following is not lyophillic
(a) Gelatin sol (b) Silver sol
(c) Sulphur sol (d) As2S3 Sol
24. Colloidion
(a) It is cellulose nitrate dispersed in alcohol
(b) Used to decreases pore size of filter paper
(c) is not a colloidal solution
(d) is lyophobic solution

25. Which of the following are multi molecular colloids


(a) Sulphur sol (b) Egg albumin in water
(c) Gold sol (d) Soap solution

SECTION – III Comprehension Type Questions

Write-up I
An iron (III) hydroxide sol. Can be made to aggregate by the addition of an ionic solution,
particularly if the solution contains anions with multiple charges (such as phosphate ions
PO 34− ). Coagulation is the process by which the dispersed phase of a colloid so made to
aggregate and thereby separate from the continous phase.
1. Colloidal sulfur particles are negatively charged with thiosulfate ions, S2 O 32 − , and other
ions on the surface of the sulphur. Indicate which of the following would be most
effective in coagulating collidal sulfur.
(a) NaCl (b) KCl
(c) MgCl2 (d) Al2(SO4)3
2. Which of the following ions is most effective in the coagulation of an arsenioussulphide
sol?
(a) K+ (b) Mg2+
3+
(c) Al (d) Ca2+
3. Which of the following ions is most effective in the coagulation of ferric hydroxide sol?
(a) Cl– (b) Br–
(c) NO −2 (d) SO 24 −

4. Which of the following solution is negatively charged?


(a) Arsenioussulphide (b) Aluminum hydroxide
(c) Ferric hydroxide (d) Silver iodide in silver nitrate solution
5. Which of the following anions will have minimum flocculation value for the ferric oxide
solution?
(a) Cl– (b) Br–
(c) SO 24 − (d) [Fe(CN)6]3–

Write-up II
If accumulation of gas on the surface of a solid occurs on account of weak Vander Waals
forces, the adsorption is termed as physical adsorption or physio-sorption. When the gas
molecules or atoms are held to the solid surface by chemical bonds, the adsorption is
termed chemical adsorption or chemisorptions. The chemical bonds may be coralent or
ionic in nature. Chemisorptionshas a rather high energy of activation and is therefore
often referred to as activated adsorption.

6. Which among the following is true?


(A) Physical adsorption is high at high temperatures while chemisorptionsis high at lower
temperatures.
(B) Chemical bonding occurs in chemisorptions
(C) The adsorption of a gas decreases with increase of pressure on a solid surface.
(D) In physical adsorption, the molecules of adsorbate are held by chemical forces
(a) B (b) A, B
(c) A (d) B, C
7. Physical adsorption is appreciable at:
(a) higher temperature (b) lower temperature
(c) room temperature (d) 100°C
8. The rate of chemi-sorption:
(a) decreases with increase of pressure
(b) is independent of pressure
(c) is maximum at one atmospheric pressure
(d) increases with increase of pressure
9. Which of the following is not a characteristic of chemi-sorption?
(a) Adsorption is irreversible
(b) ∆H is of the order of 400 kJ
(c) Adsorption is specific
(d) Adsorption increases with increase of surface area
10. Which one of the following is the correct statement?
(a) chemi-sorption is reversible in nature
(b) chemi-sorption is high at low pressure
(c) chemi-sorption depends on the nature of gas
(d) chemi-sorption does not involve activation energy

Write-up III
Enzymes are complex nitrogenous organic compounds which are produced by living
plants and animals. Enzymes are actually high molecular mass protein molecules.
Enzymes form colloidal solutions in water and are very effective catalysts. They catalyse
numerous reactions, especially those connected with natural processes. These reactions
are catalysed by enzymes. The enzymes are thus termed as bio-chemical catalysts and
phenomenon is known as bio-chemical catalysis.
11. Enzyme catalysis is an example of
(a) Auto-catalysis (b) heterogeneous catalysis
(c) homogenous catalysis (d) induced catalysis

12. Which one of the following statements is false?


(a) Enzymes are highly specific (b) Enzymes increase activation energy
(c) Enzymes require optimum pH (d) Enzymes require optimum temperature

13. Which of the following statements is incorrect?


(a) Enzymes exist in colloidal state (b) Enzymes are catalysts
(c) Enzymes can catalyse any reaction (d) Urea in an enzyme

14. The function of enzymes in the living system is to:


(a) transport oxygen (b) provide immunity
(c) catalyse biochemical reaction (d) provide energy

15. Identify the correct statement regarding enzymes:


(a) Enzymes are specific biological catalysts that can normally function at very high
temperature (T ≈ 1000 K)
(b) Enzymes are normally heterogeneous catalysts that are very specific in action
(c) Enzymes are specific biological catalysts that cannot be poisoned
(d) Enzymes are specific biological catalysts that possess well defined active site

Write-up IV
The clouds of charged particles of water disperse in air. Some of them are +vely charged,
others are negatively charged. When +vely charged clouds come closer they cause
lighting and thundering whereas when +ve and –ve charged colloids come closer they
cause heavy rain by aggregation of minute particles. It is possible to cause artificial rain
by throwing electrified S and or AgI from an aero plane and thus coagulating the mist
hanging in air.
16. When excess of AgNO3 is treated with KI solution, AgI forms
(a) +ve charged sol. (b) –vely charged sol.
(c) neutral sol. (d) true solution
17. Clouds are colloidal solution of
(a) liquid in gas (b) gas in liquid
(c) liquid in liquid (d) solid in liquid
18. AgI helps in artifical rain because
(a) It helps in condensation process (b) It helps in dispersion process
(c) It helps in coagulation (d) All of them
19. Electrical chimneys are made on the principle of
(a) Electro osmosis (b) Electro phoresis
(c) Coagulation (d) All of these
20. Smoke screens consist of
(a) Fine particles of TiO2 dispersed in air by aero planes
(b) Fine particles of AgI dispersed in air by aero planes
(c) Fine particles of Al2O3 dispersed in air by aero planes
(d) none of these

Answers to Assignments
SECTION - I

1. (a) 8. (b) 15. (a) 22. (c) 29. (c)


2. (b) 9. (b) 16. (d) 23. (c) 30. (b)
3. (c) 10. (b) 17. (a) 24. (c) 31. (a)
4. (d) 11. (b) 18. (d) 25. (c) 32. (c)
5. (d) 12. (c) 19. (a) 26. (c) 33. (b)
6. (d) 13. (c) 20. (c) 27. (b) 34. (b)
7. (b) 14. (a) 21. (d) 28. (b) 35. (b)
36. (b) 39. (b) 42. (c) 45. (a) 48. (d)
37. (c) 40. (a) 43. (b) 46. (b) 49. (d)
38. (a) 41. (c) 44. (c) 47. (a) 50. (c)

SECTION - II

1. (a), (b) 8. (a), (b), (c) 15. (a), (b), (d) 22. (a), (b), (c)
2. (a), (c) 9. (a), (b), (d) 16. (a), (b), (c) 23. (b), (c), (d)
3. (c), (d) 10. (b), (c), (d) 17. (a), (b), (d) 24. (a), (b)
4. (a), (b), (c) 11. (a), (b) 18. (a), (b), (c) 25. (a), (c)
5. (a), (b), (d) 12. (a), (b), (d) 19. (a), (b), (d)
6. (a), (b), (c) 13. (a), (b), (c) 20. (a), (b)
7. (a), (b), (d) 14. (a), (b) 21. (a), (b), (d)

SECTION - III

1. (d)
2. (c)
3. (d)
4. (b)
5. (d)
6. (a)
7. (b)
8. (d)
9. (b)
10. (c)
11. (b)
12. (b)
13. (c)
14. (c)
15. (b)
16. (a)
17. (a)
18. (a)
19. (b)
20. (a)

You might also like